CH 35: Diagnostic Studies Flashcards

pages 786-794

1
Q

A 62-year-old male presents to the emergency department with crushing substernal chest pain radiating to his left arm. The nurse anticipates the provider will order which laboratory test as the most specific indicator of myocardial infarction?

A. CK-MB
B. Troponin I
C. C-reactive protein
D. BNP

A

B. Troponin I

Rationale: Troponin I is the preferred test for detecting myocardial injury/ischemia due to its high sensitivity and specificity. Levels rise within 90 minutes and remain elevated for several days, aiding in early diagnosis and monitoring.

How well did you know this?
1
Not at all
2
3
4
5
Perfectly
2
Q

A patient with a history of heart failure presents with worsening shortness of breath and fatigue. Which lab value would the nurse expect to be elevated?

A. CK-MB
B. C-reactive protein
C. BNP
D. Troponin T

A

C. BNP

Rationale: BNP (b-type natriuretic peptide) is released in response to ventricular stretch and volume overload, making it a key biomarker in diagnosing and assessing the severity of heart failure.

How well did you know this?
1
Not at all
2
3
4
5
Perfectly
3
Q

A nurse is educating a patient about C-reactive protein (CRP). Which of the following statements are true regarding CRP? (SATA)

A. CRP is a marker of inflammation.

B. A CRP level >3 mg/dL indicates high cardiovascular risk.

C. CRP is the preferred test for diagnosing myocardial infarction.

D. Bacterial infections and inflammatory disorders can increase CRP levels.

E. CRP levels fluctuate rapidly and are unreliable.

A

A. CRP is a marker of inflammation.

B. A CRP level >3 mg/dL indicates high cardiovascular risk.

D. Bacterial infections and inflammatory disorders can increase CRP levels.

Rationale: CRP is a marker of inflammation and may predict cardiovascular risk. Levels >3 mg/dL indicate high risk. CRP increases in bacterial infections and inflammatory conditions but is not used to diagnose MI.

How well did you know this?
1
Not at all
2
3
4
5
Perfectly
4
Q

A 48-year-old male with hypertension and a BMI of 32 is undergoing a lipid panel. Which LDL level would place him at high risk for coronary artery disease?

A. 100 mg/dL
B. 125 mg/dL
C. 145 mg/dL
D. 170 mg/dL

A

D. 170 mg/dL

Rationale: LDL levels >160 mg/dL indicate a high risk for coronary artery disease. Management includes lifestyle changes and possibly statin therapy.

How well did you know this?
1
Not at all
2
3
4
5
Perfectly
5
Q

A nurse is reviewing lab results for a patient with suspected myocardial infarction. The CK-MB level is 3% of total CK. What is the nurse’s best interpretation of this finding?

A. This level confirms an MI.
B. This level is within normal limits.
C. CK-MB should be drawn 24 hours after symptom onset.
D. This result indicates heart failure.

A

B. This level is within normal limits.

Rationale: CK-MB levels <4–6% of total CK are not indicative of MI. CK-MB rises within 3–6 hours of an MI, peaks at 12–24 hours, and returns to normal within 48 hours.

How well did you know this?
1
Not at all
2
3
4
5
Perfectly
6
Q

A patient with a history of early-onset coronary artery disease is found to have an elevated homocysteine level. Which dietary recommendations should the nurse provide?

A. Increase vitamin B12 and folate intake.
B. Reduce sodium and fluid intake.
C. Consume a high-protein diet.
D. Increase simple carbohydrates.

A

A. Increase vitamin B12 and folate intake.

Rationale: Elevated homocysteine levels are linked to cardiovascular disease and are often associated with vitamin B12 and folate deficiencies.

How well did you know this?
1
Not at all
2
3
4
5
Perfectly
7
Q

Which factors can contribute to elevated triglyceride levels? (SATA)

A. Diabetes mellitus
B. Chronic kidney disease
C. Alcohol consumption
D. High HDL levels
E. High carbohydrate intake

A

A. Diabetes mellitus
B. Chronic kidney disease
C. Alcohol consumption
E. High carbohydrate intake

Rationale: Diabetes, kidney disease, alcohol intake, and a high-carb diet can raise triglycerides. High HDL levels are not associated with elevated triglycerides.

How well did you know this?
1
Not at all
2
3
4
5
Perfectly
8
Q

A 78-year-old patient has an NT-ProBNP level of 500 pg/mL. How should the nurse interpret this result?

A. This is normal for the patient’s age.
B. This level indicates severe heart failure.
C. This result suggests myocardial infarction.
D. The patient likely has an inflammatory disorder.

A

A. This is normal for the patient’s age.

Rationale: NT-ProBNP increases with age. For patients >75 years old, a level up to 449 pg/mL is considered normal.

How well did you know this?
1
Not at all
2
3
4
5
Perfectly
9
Q

A patient has an Lp(a) level of 40 mg/dL. How should the nurse interpret this finding?

A. The patient is at increased risk for atherosclerosis.
B. This level is normal.
C. The patient has a low risk for stroke.
D. The patient needs immediate anticoagulation therapy.

A

A. The patient is at increased risk for atherosclerosis.

Rationale: Lp(a) >30 mg/dL is associated with a higher risk of atherosclerosis, MI, and stroke.

How well did you know this?
1
Not at all
2
3
4
5
Perfectly
10
Q

A patient has a high-sensitivity Troponin T (hsTnT) level of 18 ng/L. How should the nurse interpret this result?

A. This is normal for a male patient.
B. This level indicates a severe MI.
C. The patient requires immediate thrombolytic therapy.
D. This is normal for a female patient.

A

A. This is normal for a male patient.

Rationale: Normal hsTnT levels are <22 ng/L in men and <14 ng/L in women.

How well did you know this?
1
Not at all
2
3
4
5
Perfectly
11
Q

A 55-year-old male with a history of hypertension and obesity has a routine lipid panel drawn. His total cholesterol level is 240 mg/dL. What is the priority intervention?

A. No intervention is needed since this value is only slightly elevated.
B. The patient should start a low-fat diet and increase physical activity.
C. The patient should take over-the-counter fish oil supplements.
D. The provider will likely prescribe a short-term course of statins.

A

B. The patient should start a low-fat diet and increase physical activity.

Rationale: A total cholesterol level >200 mg/dL increases the risk of coronary artery disease (CAD). First-line treatment includes dietary changes and exercise. Statin therapy is considered if lifestyle modifications do not lower cholesterol sufficiently.

How well did you know this?
1
Not at all
2
3
4
5
Perfectly
12
Q

Which HDL level is considered protective against cardiovascular disease?

A. 30 mg/dL
B. 45 mg/dL
C. 55 mg/dL
D. 65 mg/dL

A

D. 65 mg/dL

Rationale: HDL levels ≥60 mg/dL are considered protective against cardiovascular disease by helping remove excess cholesterol from the bloodstream.

How well did you know this?
1
Not at all
2
3
4
5
Perfectly
13
Q

A nurse is assessing a patient with dyspnea and suspected heart failure. Which conditions could cause an elevation in BNP levels? (SATA)

A. Chronic kidney disease
B. Myocardial infarction
C. Pulmonary hypertension
D. Dehydration
E. Atrial fibrillation

A

A. Chronic kidney disease
B. Myocardial infarction
C. Pulmonary hypertension
E. Atrial fibrillation

Rationale: BNP is released in response to ventricular stretch and fluid overload. It is elevated in heart failure, chronic kidney disease (due to impaired clearance), myocardial infarction (due to cardiac stress), pulmonary hypertension, and atrial fibrillation. Dehydration does not elevate BNP.

How well did you know this?
1
Not at all
2
3
4
5
Perfectly
14
Q

A 60-year-old woman with no history of heart disease undergoes routine blood work. Her high-sensitivity C-reactive protein (hs-CRP) level is 4 mg/dL. What does this finding indicate?

A. Low risk for cardiovascular disease
B. Moderate risk for cardiovascular disease
C. High risk for cardiovascular disease
D. This test is only relevant in patients with diagnosed CAD

A

C. High risk for cardiovascular disease

Rationale: An hs-CRP level >3 mg/dL indicates high cardiovascular risk. CRP is a marker of systemic inflammation and has been linked to an increased risk of myocardial infarction and stroke.

How well did you know this?
1
Not at all
2
3
4
5
Perfectly
15
Q

A patient scheduled for a fasting lipid panel asks how to prepare for the triglyceride test. Which instruction should the nurse provide?

A. “Avoid alcohol for 24 hours before the test.”
B. “You may eat a light meal before the test.”
C. “Take your usual medications, including lipid-lowering drugs.”
D. “You should exercise vigorously right before the test to get accurate results.”

A

A. “Avoid alcohol for 24 hours before the test.”

Rationale: Alcohol can significantly increase triglyceride levels and should be avoided for 24 hours before testing. Fasting for at least 8–12 hours is also recommended.

How well did you know this?
1
Not at all
2
3
4
5
Perfectly
16
Q

A 70-year-old male presents to the emergency department with intermittent chest pain that started 12 hours ago. His initial CK-MB and troponin levels are within normal limits. What is the most appropriate action?

A. Rule out myocardial infarction based on the normal lab values.
B. Obtain another set of cardiac biomarkers in 6–12 hours.
C. Prepare the patient for an immediate coronary angiography.
D. Check BNP levels to assess for heart failure.

A

B. Obtain another set of cardiac biomarkers in 6–12 hours.

Rationale: Troponin levels can take several hours to rise after myocardial injury. Serial testing is required to confirm or rule out an MI.

How well did you know this?
1
Not at all
2
3
4
5
Perfectly
17
Q

Which laboratory test is most specific for assessing vascular inflammation and increased risk for coronary artery disease?

A. C-reactive protein (CRP)
B. Lipoprotein-associated phospholipase A2 (Lp-PLA2)
C. BNP
D. CK-MB

A

B. Lipoprotein-associated phospholipase A2 (Lp-PLA2)

Rationale: Lp-PLA2 is an enzyme associated with vascular inflammation and is a specific marker for atherosclerosis and coronary artery disease (CAD). CRP is a general inflammation marker but is not as specific for CAD.

How well did you know this?
1
Not at all
2
3
4
5
Perfectly
18
Q

Which conditions can cause elevated troponin levels aside from myocardial infarction? (SATA)

A. Heart failure
B. Sepsis
C. Pulmonary embolism
D. Hypertension
E. Chronic kidney disease

A

A. Heart failure
B. Sepsis
C. Pulmonary embolism
E. Chronic kidney disease

Rationale: Troponin can be elevated in conditions that cause myocardial stress or injury, such as heart failure, sepsis, pulmonary embolism, and chronic kidney disease. Hypertension alone does not typically elevate troponin unless it leads to cardiac damage.

How well did you know this?
1
Not at all
2
3
4
5
Perfectly
19
Q

A 52-year-old female has an LDL cholesterol level of 170 mg/dL. She asks what she can do to lower her risk of heart disease. What is the most appropriate recommendation?

A. “Increase your intake of saturated fats to raise HDL levels.”
B. “Exercise regularly and reduce intake of processed foods.”
C. “Take a daily aspirin to lower cholesterol levels.”
D. “Increase simple carbohydrates to balance lipid metabolism.”

A

B. “Exercise regularly and reduce intake of processed foods.”

Rationale: Lifestyle modifications, including a diet low in saturated fats and regular exercise, are first-line treatments for high LDL cholesterol. Aspirin is used for its antiplatelet effects, not for lowering LDL.

How well did you know this?
1
Not at all
2
3
4
5
Perfectly
20
Q

A patient with chest pain has a copeptin level of 12 pmol/L. What does this finding suggest?

A. The patient has a high probability of myocardial infarction.
B. The patient’s copeptin level is normal and does not indicate MI.
C. The patient is experiencing acute heart failure.
D. The copeptin level suggests chronic kidney disease.

A

A. The patient has a high probability of myocardial infarction.

Rationale: Copeptin is a marker of arginine vasopressin release and increases rapidly in myocardial infarction. A level >10 pmol/L suggests a high likelihood of MI.

How well did you know this?
1
Not at all
2
3
4
5
Perfectly
21
Q

A 62-year-old male presents to the emergency department with chest pain that started 3 hours ago. His initial troponin level is within normal limits. What is the most appropriate next step?

A. Discharge the patient since the troponin level is normal.
B. Obtain a copeptin level to rule out myocardial infarction.
C. Repeat the troponin level in 3 hours.
D. Order an immediate cardiac stress test.

A

C. Repeat the troponin level in 3 hours.

Rationale: Troponin levels may take 4–6 hours to rise after myocardial injury. Serial measurements are needed to detect a trend in elevation.

How well did you know this?
1
Not at all
2
3
4
5
Perfectly
22
Q

Which laboratory findings are most indicative of acute coronary syndrome (ACS)? (SATA)

A. Elevated cardiac-specific troponin
B. Normal CK-MB levels
C. Increased copeptin
D. Decreased red blood cell count
E. Increased CK-MM levels

A

A. Elevated cardiac-specific troponin
C. Increased copeptin

Rationale: Elevated cardiac troponin and copeptin levels indicate myocardial injury. CK-MB may rise but is not as specific or reliable as troponin. RBC count is unrelated to ACS, and CK-MM is specific to skeletal muscle, not the heart.

How well did you know this?
1
Not at all
2
3
4
5
Perfectly
23
Q

How long after myocardial infarction can cardiac troponin levels remain elevated?

A. 2–4 days
B. 5–7 days
C. 10–14 days
D. 20–25 days

A

C. 10–14 days

Rationale: Troponin levels peak at 10–24 hours and can remain elevated for up to 10–14 days, making it useful for late diagnosis of MI.

How well did you know this?
1
Not at all
2
3
4
5
Perfectly
24
Q

A patient with suspected myocardial infarction has a normal troponin level at presentation. The provider orders a copeptin test, which is significantly elevated. What does this finding suggest?

A. The patient is at low risk for an acute cardiac event.
B. The patient likely has had a recent stroke.
C. The patient has a high probability of acute myocardial infarction.
D. The patient should be evaluated for heart failure instead of ACS.

A

C. The patient has a high probability of acute myocardial infarction.

Rationale: Copeptin is a substitute marker for arginine vasopressin and rises rapidly after MI. When combined with troponin, it increases the sensitivity for early MI detection.

How well did you know this?
1
Not at all
2
3
4
5
Perfectly
25
Q

A nurse is educating a group of nursing students about CK-MB. Which statements are correct? (SATA)

A. CK-MB is specific to the heart muscle.

B. CK-MB levels start rising 3–6 hours after MI.

C. CK-MB is the preferred biomarker for diagnosing acute MI.

D. CK-MB levels return to baseline within 12–48 hours.

E. CK-MB can be elevated in skeletal muscle injury.

A

A. CK-MB is specific to the heart muscle.
B. CK-MB levels start rising 3–6 hours after MI.
D. CK-MB levels return to baseline within 12–48 hours.

Rationale: CK-MB is specific to the heart and rises 3–6 hours after MI, peaking within 12–24 hours and returning to normal in 12–48 hours. It is no longer the preferred biomarker for diagnosing MI since troponin is more specific.

How well did you know this?
1
Not at all
2
3
4
5
Perfectly
26
Q

A patient with a history of heart failure presents with fatigue and shortness of breath. Laboratory results show a hemoglobin level of 8 g/dL. What is the priority intervention?

A. Administer oxygen therapy.
B. Initiate a blood transfusion as prescribed.
C. Encourage the patient to increase iron-rich foods.
D. Administer IV fluids to improve circulation.

A

B. Initiate a blood transfusion as prescribed.

Rationale: A hemoglobin level of 8 g/dL is significantly low, affecting oxygen delivery to tissues. A transfusion is indicated for symptomatic anemia, especially in patients with cardiovascular disease.

How well did you know this?
1
Not at all
2
3
4
5
Perfectly
27
Q

What is the primary advantage of high-sensitivity troponin assays in diagnosing myocardial infarction?

A. They provide results within 1–3 hours of myocardial injury.
B. They detect only large myocardial infarctions.
C. They are more specific than standard troponin tests.
D. They can only be used for monitoring chronic heart disease.

A

A. They provide results within 1–3 hours of myocardial injury.

Rationale: High-sensitivity troponin assays detect myocardial injury within 1–3 hours, allowing for faster diagnosis and treatment of acute MI.

How well did you know this?
1
Not at all
2
3
4
5
Perfectly
28
Q

A nurse is reviewing coagulation studies for a patient with cardiovascular disease. Which lab results are relevant in assessing clotting risk? (SATA)

A. Prothrombin time (PT)
B. Activated partial thromboplastin time (aPTT)
C. International normalized ratio (INR)
D. Hemoglobin
E. Red blood cell count

A

A. Prothrombin time (PT)
B. Activated partial thromboplastin time (aPTT)
C. International normalized ratio (INR)

Rationale: PT, aPTT, and INR measure clotting times and are crucial for patients at risk of thrombosis or bleeding. Hemoglobin and RBC count assess oxygen-carrying capacity but do not directly indicate clotting risk.

How well did you know this?
1
Not at all
2
3
4
5
Perfectly
29
Q

A patient arrives at the ED with chest pain that started 8 hours ago. The provider orders cardiac biomarkers, and the CK-MB level is slightly elevated while troponin is normal. What is the best interpretation?

A. The patient is likely experiencing a myocardial infarction.
B. The patient had a recent stroke rather than a heart attack.
C. The CK-MB elevation is likely due to skeletal muscle injury.
D. Troponin is the most specific test, so myocardial infarction is unlikely.

A

D. Troponin is the most specific test, so myocardial infarction is unlikely.

Rationale: Troponin is more specific for myocardial infarction. CK-MB can be elevated due to other causes, including skeletal muscle injury, making troponin the preferred diagnostic test.

How well did you know this?
1
Not at all
2
3
4
5
Perfectly
30
Q

Which cardiovascular condition is most likely to be associated with a low red blood cell count?

A. Hypertension
B. Heart failure
C. Peripheral artery disease
D. Aortic stenosis

A

B. Heart failure

Rationale: Chronic heart failure can lead to anemia due to reduced kidney perfusion and decreased erythropoietin production.

How well did you know this?
1
Not at all
2
3
4
5
Perfectly
31
Q

A 68-year-old female is admitted for chest pain and has an initial normal troponin level. Six hours later, her troponin level rises significantly. What is the most appropriate next step?

A. Discharge the patient with instructions for follow-up.

B. Perform an immediate coronary angiography.

C. Check CK-MB to confirm myocardial infarction.

D. Start anticoagulation therapy and continuous ECG monitoring.

A

D. Start anticoagulation therapy and continuous ECG monitoring.

Rationale: A rising troponin level strongly suggests myocardial infarction, requiring immediate management, including anticoagulation and continuous cardiac monitoring.

How well did you know this?
1
Not at all
2
3
4
5
Perfectly
32
Q

A 55-year-old male with hypertension and a history of smoking visits his primary care provider for a routine check-up. His high-sensitivity C-reactive protein (hs-CRP) level is significantly elevated. What does this finding indicate?

A. The patient is currently experiencing an acute myocardial infarction.

B. The patient has an increased risk for future cardiovascular events.

C. The patient likely has an active bacterial infection.

D. The patient is at risk for developing deep vein thrombosis (DVT).

A

B. The patient has an increased risk for future cardiovascular events.

Rationale: Elevated CRP levels are associated with systemic inflammation and atherosclerosis. A high CRP level is a predictor of future cardiovascular events rather than an indication of an active myocardial infarction.

How well did you know this?
1
Not at all
2
3
4
5
Perfectly
33
Q

A nurse is educating a patient about C-reactive protein (CRP). Which statements about CRP are accurate?

A. CRP is produced by the liver during periods of inflammation.

B. Elevated CRP levels are directly diagnostic of myocardial infarction.

C. High CRP levels are linked to an increased risk of atherosclerosis.

D. CRP levels can help predict future cardiovascular events.

E. CRP is a more specific marker for myocardial infarction than troponin.

A

A. CRP is produced by the liver during periods of inflammation.
C. High CRP levels are linked to an increased risk of atherosclerosis.
D. CRP levels can help predict future cardiovascular events.

Rationale: CRP is an inflammatory marker produced by the liver. Elevated CRP is associated with atherosclerosis and can help predict future cardiovascular events, but it is not specific for diagnosing myocardial infarction like troponin.

How well did you know this?
1
Not at all
2
3
4
5
Perfectly
34
Q

A patient hospitalized for pneumonia has an elevated CRP level. How should the nurse interpret this finding?

A. The patient is at immediate risk for a myocardial infarction.

B. The elevated CRP level is a response to systemic inflammation.

C. CRP levels are only relevant for cardiovascular conditions.

D. The patient should immediately start statin therapy.

A

B. The elevated CRP level is a response to systemic inflammation.

Rationale: CRP is an acute-phase protein that rises in response to systemic inflammation, including infections and chronic conditions like atherosclerosis. It is not specific to cardiovascular disease but can indicate increased cardiovascular risk when persistently elevated.

How well did you know this?
1
Not at all
2
3
4
5
Perfectly
35
Q

A 48-year-old male with no history of hypertension, diabetes, or hyperlipidemia presents for a routine check-up. His brother had a myocardial infarction at age 42. The physician orders a homocysteine level, which is elevated. What is the primary concern regarding this finding?

A. The patient has an acute myocardial infarction.
B. The patient has an underlying infection causing inflammation.
C. The patient has early-stage chronic kidney disease.
D. The patient has an increased risk of cardiovascular disease.

A

D. The patient has an increased risk of cardiovascular disease.

Rationale: Elevated homocysteine (Hcy) levels are associated with an increased risk for cardiovascular disease (CVD), peripheral vascular disease (PVD), and stroke. Hcy testing is particularly useful in patients with a family history of early CVD but without other common risk factors.

How well did you know this?
1
Not at all
2
3
4
5
Perfectly
36
Q

A nurse is educating a patient with elevated homocysteine levels about potential contributing factors. Which factors can lead to increased homocysteine levels? (SATA)

A. Vitamin B6 deficiency
B. Vitamin B12 deficiency
C. High dietary protein intake
D. Folate deficiency
E. Regular exercise

A

A. Vitamin B6 deficiency
B. Vitamin B12 deficiency
D. Folate deficiency

Rationale: Homocysteine is an amino acid produced during protein metabolism. Vitamin B6, B12, and folate deficiencies impair its metabolism, leading to elevated levels. High dietary protein intake does not directly contribute, and regular exercise does not increase homocysteine levels.

How well did you know this?
1
Not at all
2
3
4
5
Perfectly
37
Q

A nurse is reviewing a patient’s lab results and notes an order for a homocysteine level. Which patient history would most likely prompt this test?

A. A 38-year-old with controlled hypertension and hyperlipidemia
B. A 45-year-old with a strong family history of early cardiovascular disease
C. A 55-year-old with chronic obstructive pulmonary disease (COPD)
D. A 60-year-old with type 2 diabetes and diabetic neuropathy

A

B. A 45-year-old with a strong family history of early cardiovascular disease

Rationale: Homocysteine testing is recommended for patients with a family history of early CVD or those who have CVD without common risk factors. While hypertension and hyperlipidemia are risk factors for CVD, homocysteine testing is not typically ordered unless there is a strong family history or unexplained cardiovascular events.

How well did you know this?
1
Not at all
2
3
4
5
Perfectly
38
Q

A patient is diagnosed with elevated homocysteine levels. What dietary recommendation should the nurse make?

A. Increase intake of vitamin B6-, B12-, and folate-rich foods.

B. Increase protein consumption to balance homocysteine levels.

C. Avoid all animal products to lower homocysteine levels.

D. Consume a high-fat, low-carbohydrate diet.

A

A. Increase intake of vitamin B6-, B12-, and folate-rich foods.

Rationale: Elevated homocysteine levels are often due to deficiencies in vitamin B6, B12, and folate. Increasing dietary intake of these vitamins through sources like leafy greens, fortified cereals, eggs, and lean meats can help lower homocysteine levels and reduce cardiovascular risk.

How well did you know this?
1
Not at all
2
3
4
5
Perfectly
39
Q

A 72-year-old female with a history of hypertension and type 2 diabetes presents to the emergency department with shortness of breath, fatigue, and peripheral edema. Her vital signs are BP 150/88 mmHg, HR 98 bpm, RR 24 breaths/min, and SpO₂ 92% on room air. The physician orders a BNP test, which is significantly elevated. What does this finding most likely indicate?

A. The patient has chronic obstructive pulmonary disease (COPD) exacerbation.

B. The patient is experiencing an acute myocardial infarction.

C. The patient is experiencing heart failure exacerbation.

D. The patient has a pulmonary embolism.

A

C. The patient is experiencing heart failure exacerbation.

Rationale: BNP is primarily released from the ventricles in response to increased diastolic pressure, such as in heart failure (HF). A significantly elevated BNP level is a key indicator of HF and helps distinguish it from respiratory causes of dyspnea, such as COPD or pulmonary embolism.

How well did you know this?
1
Not at all
2
3
4
5
Perfectly
40
Q

A nurse is educating a nursing student about BNP and NT-pro-BNP. Which statements indicate correct understanding? (SATA)

A. BNP is released from the ventricles in response to increased diastolic pressure.

B. NT-pro-BNP is more specific than BNP for diagnosing heart failure.

C. BNP and NT-pro-BNP promote natriuresis and diuresis.

D. NT-pro-BNP has a longer half-life than BNP.

E. A high BNP level is diagnostic of a respiratory disorder.

A

A. BNP is released from the ventricles in response to increased diastolic pressure.
C. BNP and NT-pro-BNP promote natriuresis and diuresis.
D. NT-pro-BNP has a longer half-life than BNP.

Rationale: BNP is released from the ventricles in response to increased diastolic pressure, helping to diagnose heart failure. Both BNP and NT-pro-BNP promote natriuresis (sodium excretion) and diuresis to reduce fluid overload. NT-pro-BNP is more sensitive but less specific than BNP and has a longer half-life. BNP is used to differentiate cardiac from respiratory causes of dyspnea, not diagnose a respiratory disorder.

How well did you know this?
1
Not at all
2
3
4
5
Perfectly
41
Q

A patient’s BNP level is 950 pg/mL. What should the nurse suspect?

A. Normal cardiac function
B. Mild heart failure
C. Moderate to severe heart failure
D. Dehydration

A

C. Moderate to severe heart failure

Rationale: BNP levels correlate with the severity of heart failure. A BNP level greater than 400 pg/mL is strongly suggestive of heart failure. Levels exceeding 900 pg/mL indicate moderate to severe heart failure.

How well did you know this?
1
Not at all
2
3
4
5
Perfectly
42
Q

An 80-year-old male is admitted with worsening dyspnea and orthopnea. His NT-pro-BNP level is 600 pg/mL. The nurse understands that:

A. This value is normal for his age and rules out heart failure.

B. This value is critically elevated and suggests a myocardial infarction.

C. The patient likely has chronic kidney disease rather than heart failure.

D. NT-pro-BNP levels are higher in elderly patients, but heart failure cannot be ruled out.

A

D. NT-pro-BNP levels are higher in elderly patients, but heart failure cannot be ruled out.

Rationale: NT-pro-BNP levels naturally increase with age, making interpretation in elderly patients more complex. While levels tend to be higher, a result of 600 pg/mL does not rule out heart failure, especially in a symptomatic patient.

How well did you know this?
1
Not at all
2
3
4
5
Perfectly
43
Q

A nurse is caring for a patient with dyspnea. Which lab finding would most strongly indicate heart failure as the cause?

A. D-dimer 500 ng/mL
B. BNP 800 pg/mL
C. Troponin I 0.02 ng/mL
D. Arterial blood gas (ABG) showing respiratory alkalosis

A

B. BNP 800 pg/mL

Rationale: A BNP level above 400 pg/mL is strongly indicative of heart failure. D-dimer is used to assess for blood clots (e.g., pulmonary embolism), troponin detects myocardial injury, and ABG changes are nonspecific to HF.

How well did you know this?
1
Not at all
2
3
4
5
Perfectly
44
Q

A nurse is reviewing a patient’s elevated BNP results. Which factors can influence BNP levels and should be considered when interpreting the results? (SATA)

A. Age
B. Kidney function
C. Obesity
D. Recent steroid use
E. Hemoglobin A1c level

A

A. Age
B. Kidney function
C. Obesity

Rationale: BNP levels can be influenced by age (they naturally rise in older adults) and kidney function (renal insufficiency can lead to higher levels due to impaired clearance). Obesity is associated with lower BNP levels, which may mask heart failure. Steroid use and hemoglobin A1c levels do not significantly affect BNP interpretation.

How well did you know this?
1
Not at all
2
3
4
5
Perfectly
45
Q

A 55-year-old male with hypertension and obesity undergoes a lipid panel. His results are: total cholesterol 240 mg/dL, LDL 160 mg/dL, HDL 30 mg/dL, and triglycerides 200 mg/dL. Based on these results, the nurse identifies that the patient is at increased risk for which condition?

A. Aortic aneurysm
B. Coronary artery disease (CAD)
C. Pulmonary embolism
D. Deep vein thrombosis

A

B. Coronary artery disease (CAD)

Rationale: Elevated total cholesterol, LDL, and triglycerides, along with low HDL, are strong risk factors for CAD. LDL contributes to plaque formation, while HDL plays a protective role.

How well did you know this?
1
Not at all
2
3
4
5
Perfectly
46
Q

A nurse is teaching a patient about different lipoproteins and their effects on cardiovascular health. Which statements indicate correct understanding? (SATA)

A. LDL cholesterol contributes to plaque formation in the arteries.

B. HDL cholesterol helps remove excess cholesterol from the bloodstream.

C. VLDL is primarily composed of triglycerides.

D. Chylomicrons transport dietary fats from the intestines to the liver.

E. An increased HDL level is associated with an increased risk of CAD.

A

A. LDL cholesterol contributes to plaque formation in the arteries.
B. HDL cholesterol helps remove excess cholesterol from the bloodstream.
C. VLDL is primarily composed of triglycerides.
D. Chylomicrons transport dietary fats from the intestines to the liver.

Rationale: LDL contributes to atherosclerosis, HDL helps remove cholesterol from tissues, VLDL is primarily triglycerides, and chylomicrons transport dietary fat. Increased HDL is protective, reducing CAD risk.

How well did you know this?
1
Not at all
2
3
4
5
Perfectly
47
Q

A patient has an LDL level of 170 mg/dL. The nurse recognizes that this level places the patient at:

A. Low risk for CAD
B. Moderate risk for CAD
C. High risk for CAD
D. No increased risk for CAD

A

C. High risk for CAD

Rationale: An LDL level of 160 mg/dL or higher is considered a major risk factor for CAD.

How well did you know this?
1
Not at all
2
3
4
5
Perfectly
48
Q

A 45-year-old female presents for a routine checkup. Her total cholesterol is 190 mg/dL, LDL is 110 mg/dL, HDL is 65 mg/dL, and triglycerides are 120 mg/dL. Based on these values, the nurse determines that her risk for CAD is:

A. Increased due to her high total cholesterol
B. Increased due to her high triglycerides
C. Low due to her high HDL level
D. High due to her LDL level

A

C. Low due to her high HDL level

Rationale: A high HDL level (above 60 mg/dL) is protective and reduces the risk of CAD, even if total cholesterol or LDL levels are borderline.

How well did you know this?
1
Not at all
2
3
4
5
Perfectly
49
Q

A patient’s total cholesterol is 240 mg/dL, and HDL is 40 mg/dL. The nurse calculates the cholesterol-to-HDL ratio as:

A. 3
B. 4
C. 5
D. 6

A

D. 6

Rationale: The cholesterol-to-HDL ratio is calculated as total cholesterol ÷ HDL. A ratio above 5 increases CAD risk.

How well did you know this?
1
Not at all
2
3
4
5
Perfectly
50
Q

Which factors contribute to elevated LDL and triglyceride levels? (SATA)

A. Obesity
B. Smoking
C. Sedentary lifestyle
D. High-fiber diet
E. Excessive alcohol intake

A

A. Obesity
B. Smoking
C. Sedentary lifestyle
E. Excessive alcohol intake

Rationale: Obesity, smoking, lack of exercise, and alcohol intake raise LDL and triglycerides. A high-fiber diet lowers LDL levels.

How well did you know this?
1
Not at all
2
3
4
5
Perfectly
51
Q

A nurse is educating a patient about lipoprotein (a) [Lp(a)]. What statement indicates correct understanding?

A. “High Lp(a) levels reduce my risk of heart disease.”

B. “Lp(a) has no connection to atherosclerosis.”

C. “Increased Lp(a) levels are linked to an increased risk of atherosclerosis.”

D. “Lp(a) is only a concern for men and not for women.”

A

C. “Increased Lp(a) levels are linked to an increased risk of atherosclerosis.”

Rationale: High Lp(a) levels are a risk factor for atherosclerosis and CAD, especially in women.

How well did you know this?
1
Not at all
2
3
4
5
Perfectly
52
Q

A patient with no history of heart disease undergoes lab testing and has significantly elevated lipoprotein-associated phospholipase A2 (Lp-PLA2). What does this result indicate?

A. Increased risk for CAD
B. Decreased risk for CAD
C. No clinical significance
D. Increased blood clotting ability

A

A. Increased risk for CAD

Rationale: Lp-PLA2 is an inflammatory enzyme found in atherosclerotic plaques. High levels indicate increased CAD risk.

How well did you know this?
1
Not at all
2
3
4
5
Perfectly
53
Q

Which dietary habit would most effectively help lower LDL cholesterol?

A. Increasing red meat intake
B. Replacing saturated fats with unsaturated fats
C. Avoiding all carbohydrates
D. Increasing sodium intake

A

B. Replacing saturated fats with unsaturated fats

Rationale: Unsaturated fats (found in fish, nuts, and olive oil) help lower LDL, while saturated fats (in red meat and dairy) increase LDL.

How well did you know this?
1
Not at all
2
3
4
5
Perfectly
54
Q

A patient with an LDL level of 170 mg/dL asks why their doctor ordered an apolipoprotein B (apo B) test. The nurse explains that apo B:

A. Is only used to assess liver function.
B. Has no relation to cholesterol levels.
C. Is the main protein found in HDL.
D. Is the major protein in LDL and strongly predicts CAD risk.

A

D. Is the major protein in LDL and strongly predicts CAD risk.

Rationale: Apo B is the primary protein in LDL and is a stronger predictor of CAD than LDL alone.

How well did you know this?
1
Not at all
2
3
4
5
Perfectly
55
Q

Which statements correctly describe lipoproteins? (SATA)

A. Chylomicrons transport dietary fat from the intestines.

B. LDL is primarily composed of triglycerides.

C. HDL removes excess cholesterol from tissues.

D. VLDL is rich in endogenous triglycerides.

E. Increased HDL is protective against CAD.

A

A. Chylomicrons transport dietary fat from the intestines.
C. HDL removes excess cholesterol from tissues.
D. VLDL is rich in endogenous triglycerides.
E. Increased HDL is protective against CAD.

Rationale: LDL is mostly cholesterol, not triglycerides. The other statements are correct.

How well did you know this?
1
Not at all
2
3
4
5
Perfectly
56
Q

Apo A-I is a major component of which lipoprotein?

A. LDL
B. VLDL
C. HDL
D. Chylomicrons

A

C. HDL

Rationale: Apo A-I is the main protein in HDL and helps remove cholesterol from tissues, reducing CAD risk.

How well did you know this?
1
Not at all
2
3
4
5
Perfectly
57
Q

A patient is scheduled for a lipid panel. The nurse instructs them to:

A. Avoid food for 12 hours before the test.
B. Consume a high-fat meal the night before testing.
C. Avoid water intake before the test.
D. Take lipid-lowering medications right before the test.

A

A. Avoid food for 12 hours before the test.

Rationale: A fasting lipid panel requires at least 8-12 hours of fasting for accurate triglyceride and LDL measurements.

How well did you know this?
1
Not at all
2
3
4
5
Perfectly
58
Q

What HDL level is considered protective against CAD?

A. <40 mg/dL
B. 40-50 mg/dL
C. >60 mg/dL
D. 50-60 mg/dL

A

C. >60 mg/dL

Rationale: HDL levels above 60 mg/dL are protective against CAD.

How well did you know this?
1
Not at all
2
3
4
5
Perfectly
59
Q

Which ratio most strongly predicts CAD risk?

A. LDL:HDL ratio
B. Triglycerides:LDL ratio
C. HDL:VLDL ratio
D. Total cholesterol:HDL ratio

A

D. Total cholesterol:HDL ratio

Rationale: The total cholesterol to HDL ratio is a key predictor of CAD risk.

How well did you know this?
1
Not at all
2
3
4
5
Perfectly
60
Q

A nurse is preparing a patient for a 12-lead ECG. Which statement by the nurse is appropriate?

A. “You may feel a slight shock when the test begins.”
B. “You should remain still during the test to prevent artifacts.”
C. “This test requires you to perform mild exercises during recording.”
D. “You must fast for 8 hours before the test.”

A

B. “You should remain still during the test to prevent artifacts.”

Rationale: A 12-lead ECG requires the patient to remain still to reduce motion artifact, which can interfere with accurate recording. The test is painless and does not require fasting or physical activity.

How well did you know this?
1
Not at all
2
3
4
5
Perfectly
61
Q

A patient is scheduled for Holter monitoring. Which statement by the patient indicates a need for further teaching?

A. “I will keep a diary of my activities and symptoms.”
B. “I should avoid taking a shower while wearing the monitor.”
C. “I can remove the electrodes at night for comfort.”
D. “I will push the event marker when I feel symptoms.”

A

C. “I can remove the electrodes at night for comfort.”

Rationale: The electrodes must remain in place continuously during the 24–48 hour monitoring period to ensure accurate data collection.

How well did you know this?
1
Not at all
2
3
4
5
Perfectly
62
Q

A patient using an ECG event monitor asks how it works. The nurse explains that the device:

A. Continuously records the heart rhythm for 24 hours.

B. Automatically transmits ECG data to the provider every hour.

C. Requires hospitalization for continuous monitoring.

D. Records heart rhythms only when activated by the patient during symptoms.

A

D. Records heart rhythms only when activated by the patient during symptoms.

Rationale: Event monitors capture transient cardiac events by recording when activated, which allows detection of infrequent arrhythmias.

How well did you know this?
1
Not at all
2
3
4
5
Perfectly
63
Q

The nurse is reviewing a patient’s ECG and notices ST-segment elevation. This finding is most indicative of:

A. Atrial fibrillation
B. Myocardial infarction
C. Ventricular tachycardia
D. Sinus bradycardia

A

B. Myocardial infarction

Rationale: ST-segment elevation is a hallmark of acute myocardial infarction due to myocardial ischemia.

How well did you know this?
1
Not at all
2
3
4
5
Perfectly
64
Q

Which of the following is the primary reason for performing a signal-averaged ECG (SAECG)?

A. To assess the electrical activity of the sinoatrial node
B. To detect ventricular late potentials and risk for ventricular tachycardia
C. To evaluate the effectiveness of antiarrhythmic medications
D. To diagnose pericarditis

A

B. To detect ventricular late potentials and risk for ventricular tachycardia

Rationale: SAECG is used to identify late potentials, which indicate a risk for ventricular tachycardia.

How well did you know this?
1
Not at all
2
3
4
5
Perfectly
65
Q

A patient using a Holter monitor reports skin irritation under the electrodes. The nurse should instruct the patient to:

A. Remove the electrodes and reapply them later
B. Continue wearing the monitor and report irritation after 24 hours
C. Apply an antihistamine cream under the electrodes
D. Notify the provider, as the electrodes may need to be repositioned

A

D. Notify the provider, as the electrodes may need to be repositioned

Rationale: Skin irritation is a common issue, and the provider may need to adjust electrode placement while maintaining accurate monitoring.

How well did you know this?
1
Not at all
2
3
4
5
Perfectly
66
Q

A patient with chest pain is scheduled for an ECG event monitor. Which patient statement requires further teaching?

A. “This device continuously records my heart rhythm for 24 hours.”
B. “I should press the button when I feel symptoms.”
C. “I need to keep good skin contact with the device.”
D. “My ECG recordings can be transmitted to my provider.”

A

A. “This device continuously records my heart rhythm for 24 hours.”

Rationale: Unlike Holter monitors, ECG event monitors do not record continuously; they capture cardiac events when activated.

How well did you know this?
1
Not at all
2
3
4
5
Perfectly
67
Q

A patient undergoing Holter monitoring asks why they must keep a diary. The nurse explains that:

A. The diary helps correlate symptoms with ECG changes.
B. The diary is required for insurance purposes.
C. The diary replaces the need for ECG analysis.
D. The diary helps the patient remain calm during monitoring.

A

A. The diary helps correlate symptoms with ECG changes.

Rationale: A symptom diary allows healthcare providers to match patient-reported symptoms with ECG abnormalities.

How well did you know this?
1
Not at all
2
3
4
5
Perfectly
68
Q

The nurse is educating a patient about a 12-lead ECG. Which of the following should the nurse include in the teaching?

A. “This test requires mild electrical stimulation of the heart.”
B. “You should limit fluid intake before the test.”
C. “This test will record your heart’s electrical activity from multiple angles.”
D. “You will need to perform light exercise before the test.”

A

C. “This test will record your heart’s electrical activity from multiple angles.”

Rationale: A 12-lead ECG provides a comprehensive view of electrical activity from different perspectives.

How well did you know this?
1
Not at all
2
3
4
5
Perfectly
69
Q

A patient reports palpitations that occur only a few times a week. The provider orders an event monitor instead of a Holter monitor because:

A. Event monitors are more accurate than Holter monitors.
B. Holter monitors are only used for severe cardiac conditions.
C. Event monitors can capture infrequent arrhythmias over a longer period.
D. Holter monitors require hospitalization.

A

C. Event monitors can capture infrequent arrhythmias over a longer period.

Rationale: Event monitors are useful for detecting intermittent arrhythmias that may not occur within a 24-hour Holter monitoring period.

How well did you know this?
1
Not at all
2
3
4
5
Perfectly
70
Q

A nurse is applying electrodes for a 12-lead ECG. Where should the nurse place the limb leads?

A. On the bony areas of the extremities
B. Over large muscle groups to reduce artifact
C. On the patient’s chest near the heart
D. At the patient’s waistline

A

B. Over large muscle groups to reduce artifact

Rationale: Limb leads should be placed over muscle to ensure stable electrode contact and reduce artifact.

How well did you know this?
1
Not at all
2
3
4
5
Perfectly
71
Q

A nurse instructs a patient to avoid bathing while wearing a Holter monitor. This instruction is given because:

A. Water can interfere with the electrical activity of the heart.
B. Wet skin reduces the effectiveness of the electrodes.
C. The monitor is waterproof but the electrodes are not.
D. Water exposure may erase recorded data.

A

B. Wet skin reduces the effectiveness of the electrodes.

Rationale: Moisture can cause electrodes to detach, leading to inaccurate data collection.

How well did you know this?
1
Not at all
2
3
4
5
Perfectly
72
Q

A patient with a suspected arrhythmia has a signal-averaged ECG ordered. The patient asks why this test is needed. The nurse explains that this test:

A. Evaluates the effectiveness of a pacemaker.
B. Identifies small electrical signals that may trigger arrhythmias.
C. Measures the heart’s pumping ability.
D. Provides real-time cardiac imaging.

A

B. Identifies small electrical signals that may trigger arrhythmias.

Rationale: SAECG detects ventricular late potentials that increase the risk for ventricular tachycardia.

How well did you know this?
1
Not at all
2
3
4
5
Perfectly
73
Q

A patient with an event monitor is instructed to press the event button when symptoms occur. The nurse should also advise the patient to:

A. Call 911 if experiencing severe chest pain or dizziness.
B. Remove the monitor when sleeping.
C. Apply lotion to prevent skin irritation.
D. Exercise regularly while wearing the monitor.

A

A. Call 911 if experiencing severe chest pain or dizziness.

Rationale: Patients should seek emergency care if experiencing serious cardiac symptoms.

How well did you know this?
1
Not at all
2
3
4
5
Perfectly
74
Q

Which of the following patients is the best candidate for Holter monitoring?

A. A patient with daily chest pain episodes
B. A patient with occasional palpitations every few weeks
C. A patient with continuous ST-segment elevation
D. A patient with a diagnosed arrhythmia requiring real-time monitoring

A

A. A patient with daily chest pain episodes

Rationale: Holter monitoring is most useful for detecting frequent but intermittent arrhythmias over a 24–48 hour period.

How well did you know this?
1
Not at all
2
3
4
5
Perfectly
75
Q

A nurse is preparing a patient for a 6-minute walk test. Which statement by the patient indicates the need for further teaching?

a. “I should wear comfortable shoes for this test.”
b. “I will try to walk as quickly as possible during the test.”
c. “I should avoid taking my oxygen with me during the test, even though I use it daily.”
d. “This test will help assess my ability to perform daily activities.”

A

c. “I should avoid taking my oxygen with me during the test, even though I use it daily.”

Rationale: Patients who use oxygen routinely should carry or pull it with them during the test to ensure adequate oxygenation.

How well did you know this?
1
Not at all
2
3
4
5
Perfectly
76
Q

A nurse is educating a patient scheduled for an exercise stress test. Which instruction should the nurse include?

a. “Take your beta-blocker medication as scheduled before the test.”
b. “You should refrain from smoking or exercising for at least 12 hours before the test.”
c. “Avoid caffeine for at least 24 hours before the test.”
d. “You should wear tight-fitting clothing to provide support during the test.”

A

c. “Avoid caffeine for at least 24 hours before the test.”

Rationale: Caffeine can interfere with test results by affecting heart rate and should be avoided for 24 hours before the stress test.

How well did you know this?
1
Not at all
2
3
4
5
Perfectly
77
Q

A nurse is monitoring a patient during an exercise stress test. Which finding requires the nurse to stop the test immediately?

a. Blood pressure increases from 120/80 mmHg to 140/85 mmHg
b. The patient reports mild fatigue
c. ST-segment depression on ECG
d. Heart rate increases gradually in response to exercise

A

c. ST-segment depression on ECG

Rationale: ST-segment depression is a sign of myocardial ischemia, indicating inadequate blood flow to the heart muscle. The test should be stopped immediately.

How well did you know this?
1
Not at all
2
3
4
5
Perfectly
78
Q

A patient undergoing a 6-minute walk test reports dyspnea and lightheadedness. What is the priority nursing action?

a. Encourage the patient to keep walking but slow down.
b. Have the patient sit down and assess vital signs.
c. Administer oxygen immediately.
d. Document the symptoms and allow the patient to rest before continuing.

A

b. Have the patient sit down and assess vital signs.

Rationale: Safety is the priority. The nurse should stop the test, assess the patient’s condition, and determine the need for further intervention.

How well did you know this?
1
Not at all
2
3
4
5
Perfectly
79
Q

A patient scheduled for an exercise stress test states, “I feel fine. I don’t think I need this test.” What is the nurse’s best response?

a. “This test is only needed if you are experiencing symptoms.”

b. “We can cancel the test if you prefer.”

c. “The test can help identify heart problems even if you do not have symptoms.”

d. “The test is mainly for patients who have already been diagnosed with heart disease.”

A

c. “The test can help identify heart problems even if you do not have symptoms.”

Rationale: Stress testing is useful in detecting underlying cardiac conditions that may not present with obvious symptoms.

How well did you know this?
1
Not at all
2
3
4
5
Perfectly
80
Q

The nurse is reviewing a patient’s medications before an exercise stress test. Which medication should the nurse question?

a. Aspirin
b. Beta-blocker
c. Calcium supplement
d. Multivitamin

A

b. Beta-blocker

Rationale: Beta-blockers may be held for 24 hours before a stress test because they can blunt the heart rate response, affecting test results.

How well did you know this?
1
Not at all
2
3
4
5
Perfectly
81
Q

A nurse is explaining noninvasive hemodynamic monitoring to a patient’s family. Which statement by the family member indicates a need for further clarification?

a. “This test measures blood pressure continuously without inserting a catheter.”

b. “The test provides repeated measurements of stroke volume and cardiac output.”

c. “This test has minimal to no risk for the patient.”

d. “This procedure is only performed in the operating room.”

A

d. “This procedure is only performed in the operating room.”

Rationale: Noninvasive hemodynamic monitoring is commonly used in perioperative settings, critical care units, and emergency departments, not just in the OR.

How well did you know this?
1
Not at all
2
3
4
5
Perfectly
82
Q

A patient scheduled for an exercise stress test asks how their target heart rate is determined. What is the nurse’s best response?

a. “It is based on your age and calculated by subtracting your age from 220.”

b. “It is determined by your resting heart rate and blood pressure.”

c. “We increase the treadmill speed until you feel very tired.”

d. “The cardiologist will decide based on your medical history.”

A

a. “It is based on your age and calculated by subtracting your age from 220.”

Rationale: The predicted peak heart rate for a stress test is determined by subtracting the patient’s age from 220.

How well did you know this?
1
Not at all
2
3
4
5
Perfectly
83
Q

A nurse is caring for a patient who just completed a treadmill stress test. Which post-procedure finding would require immediate intervention?

a. The patient’s blood pressure remains slightly elevated.
b. The patient complains of mild leg fatigue.
c. The patient reports feeling a little thirsty.
d. The patient has persistent ST-segment changes on ECG.

A

d. The patient has persistent ST-segment changes on ECG.

Rationale: Persistent ST-segment changes indicate ongoing myocardial ischemia, requiring immediate medical evaluation.

How well did you know this?
1
Not at all
2
3
4
5
Perfectly
84
Q

A patient with a history of heart disease is scheduled for a 6-minute walk test. What should the nurse do first?

a. Ensure the patient wears a cardiac monitor.
b. Obtain a baseline set of vital signs.
c. Encourage the patient to walk as fast as possible.
d. Administer a beta-blocker before the test.

A

b. Obtain a baseline set of vital signs.

Rationale: Baseline vital signs provide a reference point to evaluate the patient’s response to the test and detect any abnormalities.

How well did you know this?
1
Not at all
2
3
4
5
Perfectly
85
Q

A nurse is preparing a patient for a chest x-ray. Which action should the nurse take before the procedure?

a. Instruct the patient to wear a loose-fitting shirt for easy removal.

b. Ask the patient if they have had any recent x-rays and check for possible pregnancy.

c. Remove any jewelry or metal objects on the patient’s legs.

d. Ensure the patient has an empty bladder before the procedure.

A

b. Ask the patient if they have had any recent x-rays and check for possible pregnancy.

Rationale: Before performing a chest x-ray, it is important to assess the frequency of recent x-rays and check for pregnancy due to the potential risks associated with radiation exposure.

How well did you know this?
1
Not at all
2
3
4
5
Perfectly
86
Q

A nurse is preparing a patient for a cardiac CT with IV contrast. What should the nurse do before the procedure?

a. Ask the patient about allergies, especially to iodine or contrast materials.
b. Instruct the patient to wear a metal necklace for comfort during the procedure.
c. Restrict the patient’s fluid intake for 12 hours before the procedure.
d. Ensure the patient is fasting for at least 6 hours before the procedure.

A

a. Ask the patient about allergies, especially to iodine or contrast materials.

Rationale: It is important to assess for allergies to iodine or contrast materials, as these can cause serious allergic reactions during a CT with IV contrast.

How well did you know this?
1
Not at all
2
3
4
5
Perfectly
87
Q

A patient is undergoing a cardiac CT with electron beam CT (EBCT) for coronary artery calcification. What is the primary purpose of this procedure?

a. To assess heart function during exercise
b. To identify signs of myocardial ischemia
c. To evaluate coronary artery calcification for heart disease risk
d. To visualize the heart’s electrical activity during stress

A

c. To evaluate coronary artery calcification for heart disease risk

Rationale: EBCT is primarily used to assess coronary artery calcification, which is an important indicator of heart disease risk.

How well did you know this?
1
Not at all
2
3
4
5
Perfectly
88
Q

After a chest x-ray, the nurse notices that the patient’s lung fields show signs of pulmonary edema. What is the most likely diagnosis?

a. Congestive heart failure
b. Chronic obstructive pulmonary disease (COPD)
c. Asthma exacerbation
d. Pneumonia

A

a. Congestive heart failure

Rationale: Pulmonary edema is a common finding in congestive heart failure due to the accumulation of fluid in the lungs as a result of left-sided heart failure.

How well did you know this?
1
Not at all
2
3
4
5
Perfectly
89
Q

A nurse is reviewing the results of a cardiac CT scan. The patient has a calcium score that is below the threshold level. What is the most appropriate action?

a. Immediate referral for invasive coronary angiography
b. No further action is required unless the patient develops symptoms
c. Initiate pharmacological treatment for coronary artery disease
d. Continue to monitor the patient with periodic ECG tests

A

b. No further action is required unless the patient develops symptoms

Rationale: A low calcium score suggests minimal or no coronary artery disease, and no immediate intervention is needed unless symptoms arise.

How well did you know this?
1
Not at all
2
3
4
5
Perfectly
90
Q

A patient is scheduled for a cardiac CT with IV contrast to evaluate coronary circulation. What should the nurse assess before administering the contrast?

a. The patient’s ability to tolerate exercise during the test
b. The patient’s renal function, especially creatinine levels
c. The patient’s blood pressure to determine if they are hypertensive
d. The patient’s history of heart disease to determine test relevance

A

b. The patient’s renal function, especially creatinine levels

Rationale: IV contrast can affect kidney function, so assessing renal function (especially creatinine levels) is important to prevent contrast-induced nephropathy.

How well did you know this?
1
Not at all
2
3
4
5
Perfectly
91
Q

A patient undergoing a chest x-ray is concerned about radiation exposure. What should the nurse explain?

a. “Radiation exposure is minimized, and lead shielding is used to protect areas not being imaged.”

b. “The procedure uses very high doses of radiation, so it is safe to have done regularly.”

c. “The radiation from the x-ray is enough to cause significant harm if done more than once a year.”

d. “The procedure is completely free of risk, and no precautions are needed.”

A

a. a. “Radiation exposure is minimized, and lead shielding is used to protect areas not being imaged.”

Rationale: Radiation exposure is minimized during x-rays, and lead shielding is used to protect areas not being imaged from unnecessary exposure.

How well did you know this?
1
Not at all
2
3
4
5
Perfectly
92
Q

A patient with atypical cardiac symptoms is being evaluated with a cardiac CT. Which statement by the nurse is most appropriate before the procedure?

a. “You will need to remain still for a prolonged period while the images are taken.”

b. “You may be asked to walk on a treadmill during this test.”

c. “This test uses a scanning electron beam to measure coronary artery calcification.”

d. “The contrast used in this procedure may make you feel very hot and flushed.”

A

d. “The contrast used in this procedure may make you feel very hot and flushed.”

Rationale: The IV contrast used in cardiac CT can cause a warm, flushed sensation, which is a common and benign side effect.

93
Q

A patient is scheduled for a cardiovascular magnetic resonance imaging (CMRI) to evaluate their heart function. What should the nurse assess before the procedure?

a. The patient’s ability to tolerate physical activity during the test
b. The patient’s renal function, allergies, and pregnancy status
c. The patient’s history of hypertension and current blood pressure
d. The patient’s ability to lie flat for extended periods of time

A

b. The patient’s renal function, allergies, and pregnancy status

Rationale: Before a CMRI, the nurse must assess renal function, allergies, and pregnancy status to ensure the patient’s safety during the procedure, especially since it involves contrast materials.

94
Q

During a cardiovascular magnetic resonance imaging (CMRI) procedure, what is the nurse’s role in ensuring the patient’s safety and comfort?

a. Ensuring that the patient remains still to avoid image distortion

b. Administering pain medications to prevent discomfort from the procedure

c. Continuously monitoring vital signs throughout the procedure

d. Applying pressure to prevent any potential bleeding at the catheter insertion site

A

a. Ensuring that the patient remains still to avoid image distortion

Rationale: The patient must lie completely still during the CMRI to obtain clear images, as any movement can distort the results.

95
Q

A patient is scheduled for coronary CT angiography (CTA) to evaluate coronary artery disease (CAD). What should the nurse inform the patient before the procedure?

a. “You will need to refrain from eating for 12 hours prior to the test.”

b. “You will need to have your blood pressure checked during the test.”

c. “You may need to fast for several hours, and a β-blocker may be given to control your heart rate.”

d. “It is important that you walk on a treadmill to prepare for the test.”

A

c. “You may need to fast for several hours, and a β-blocker may be given to control your heart rate.”

Rationale: The patient may need to fast prior to the procedure, and a β-blocker may be administered to control heart rate, which is essential for accurate imaging during CTA.

96
Q

A nurse is preparing a patient for a coronary CT angiography (CTA) procedure. What is an important nursing responsibility before the test?

a. Ensure that the patient is hydrated to help with contrast excretion
b. Obtain IV access and assess for a contrast allergy
c. Provide instructions to the patient about exercise stress testing before the procedure
d. Administer analgesic medications for comfort during the procedure

A

b. Obtain IV access and assess for a contrast allergy

Rationale: Before the CTA, the nurse must ensure IV access is established for contrast injection and assess for any contrast allergies to avoid allergic reactions.

97
Q

A patient is undergoing cardiovascular magnetic resonance imaging (CMRI). What contraindication should the nurse be aware of?

a. The patient’s history of diabetes
b. The presence of implanted metallic devices or metal fragments
c. The patient’s recent history of heart attack
d. The patient’s elevated blood pressure

A

b. The presence of implanted metallic devices or metal fragments

Rationale: CMRI is contraindicated for patients with implanted metallic devices or metal fragments, as the magnetic field can interfere with these devices, posing a risk to the patient.

98
Q

A patient undergoing coronary CT angiography (CTA) requires a regular heart rhythm for accurate testing. What should the nurse do if the patient’s heart rate becomes irregular during the test?

a. Administer a sedative to calm the patient and reduce anxiety

b. Ask the patient to perform deep breathing exercises to control their heart rate

c. Resume the procedure and proceed with the irregular rhythm for accurate results

d. Notify the healthcare provider to determine the need for further intervention

A

d. Notify the healthcare provider to determine the need for further intervention

Rationale: An irregular heart rhythm can interfere with the accuracy of CTA results. The nurse should notify the healthcare provider to determine the appropriate action, which may include further interventions or rescheduling the test.

99
Q

A nurse is preparing a patient for a pharmacologic echocardiogram using adenosine. What is the primary purpose of this test?

a. To assess for wall motion abnormalities in patients unable to exercise

b. To measure cardiac output and determine heart rate response during exercise

c. To evaluate coronary artery disease in patients with atypical symptoms

d. To determine the presence of congenital heart defects

A

a. To assess for wall motion abnormalities in patients unable to exercise

Rationale: A pharmacologic echocardiogram uses medications like adenosine to simulate the stress response in patients who cannot undergo exercise stress testing, allowing assessment of wall motion abnormalities.

100
Q

Which of the following actions is a priority before conducting a pharmacologic echocardiogram?

a. Provide a light meal for the patient to eat before the procedure

b. Obtain baseline vital signs and ensure IV access for drug administration

c. Administer an analgesic to prevent discomfort during the procedure

d. Ask the patient to hold their breath for the duration of the procedure

A

b. Obtain baseline vital signs and ensure IV access for drug administration

Rationale: Baseline vital signs should be obtained and IV access established before administering pharmacologic agents such as adenosine or dobutamine to ensure patient safety during the procedure.

101
Q

What is an important nursing responsibility when preparing a patient for exercise nuclear imaging?

a. Ensure the patient has not eaten for at least 12 hours before the procedure

b. Instruct the patient to refrain from caffeine consumption for 12 hours prior

c. Advise the patient to eat a heavy meal to prepare for the procedure

d. Instruct the patient to perform moderate exercise before the test

A

b. Instruct the patient to refrain from caffeine consumption for 12 hours prior

Rationale: Caffeine can interfere with the imaging results by affecting the heart’s response during the test, so it is important for the patient to avoid caffeine 12 hours before the procedure.

102
Q

A patient undergoing magnetic resonance angiography (MRA) asks about the procedure. What is the nurse’s primary concern before the procedure?

a. Ensuring that the patient has not eaten for 6 hours prior to the test

b. Checking for allergies to contrast medium and the presence of implanted metallic devices

c. Assessing the patient’s ability to tolerate physical activity during the test

d. Monitoring the patient for symptoms of dizziness during the procedure

A

b. Checking for allergies to contrast medium and the presence of implanted metallic devices

Rationale: MRA involves the use of gadolinium contrast medium, and it is contraindicated for patients with allergies to this substance or implanted metallic devices, so the nurse must assess for these factors beforehand.

103
Q

During a nuclear imaging procedure, the patient is required to lie still with their arms extended overhead. What should the nurse explain to the patient?

a. The procedure will involve frequent interruptions to administer medication

b. The radioactive isotopes used will remain in the patient’s body for several hours

c. The arms must be held in position to minimize artifacts and improve image quality

d. The procedure will be completed in a few minutes, and the patient can resume normal activities immediately

A

c. The arms must be held in position to minimize artifacts and improve image quality

Rationale: Holding the arms in position during the scan helps reduce artifacts in the imaging and ensures the best possible image quality.

104
Q

A patient is scheduled for a multigated acquisition (MUGA) scan to assess cardiac function. What is a key nursing responsibility during this procedure?

a. Administering sedatives to ensure the patient remains relaxed
b. Monitoring the patient for ECG changes during the procedure
c. Instructing the patient to avoid any physical movement during the test
d. Explaining that the test will involve exercise to assess the heart’s response

A

b. Monitoring the patient for ECG changes during the procedure

Rationale: The MUGA scan requires continuous ECG monitoring during the procedure to obtain accurate timing of cardiac events and ensure proper image acquisition.

105
Q

Which of the following is a contraindication for a pharmacologic nuclear imaging procedure using adenosine or dipyridamole?

a. Previous history of coronary artery bypass surgery
b. Use of β-blockers within the past 24 hours
c. Pregnancy or potential pregnancy
d. Recent use of aspirin for pain relief

A

c. Pregnancy or potential pregnancy

Rationale: Pregnant patients or those who could be pregnant should avoid pharmacologic nuclear imaging procedures due to potential risks to the fetus from radiation exposure and the medications used during the test.

106
Q

A nurse is preparing to administer a contrast medium for an MRI. What should the nurse assess before the procedure?

a. The patient’s ability to exercise and tolerate stress

b. The patient’s history of allergies and metal implants

c. The patient’s blood glucose level to prevent hypoglycemia

d. The patient’s ability to remain calm and relaxed during the procedure

A

b. The patient’s history of allergies and metal implants

Rationale: It is essential to assess for allergies to contrast medium and the presence of metal implants, as MRI procedures can be dangerous for patients with certain metallic implants.

107
Q

During a pharmacologic nuclear imaging procedure, the nurse should observe the patient for which of the following side effects?

a. Chest pain and palpitations
b. Bradycardia and hypotension
c. Dyspnea, dizziness, and nausea
d. Hypertension and tachycardia

A

c. Dyspnea, dizziness, and nausea

Rationale: These are common side effects of vasodilators like adenosine, dipyridamole, and regadenoson used during pharmacologic nuclear imaging. The nurse should monitor for these symptoms and be prepared to administer aminophylline if needed.

108
Q

A patient undergoing a pharmacologic echocardiogram complains of dizziness and nausea. What should the nurse do first?

a. Administer oxygen via nasal cannula
b. Notify the healthcare provider immediately
c. Administer aminophylline if prescribed
d. Reassure the patient that these symptoms are normal and will resolve soon

A

c. Administer aminophylline if prescribed

Rationale: Aminophylline is used to reverse the side effects of dipyridamole (dyspnea, dizziness, and nausea) and should be administered if ordered by the healthcare provider.

109
Q

The nurse is preparing a patient for a contrast echocardiogram. What instruction should be given to the patient before the procedure?

a. “You should not eat or drink for 12 hours before the procedure.”

b. “You will need to take a laxative to prepare for the test.”

c. “You will be required to exercise during the test to increase heart rate.”

d. “You may feel some pressure and mechanical movement from the transducer during the procedure.”

A

d. “You may feel some pressure and mechanical movement from the transducer during the procedure.”

Rationale: Patients undergoing echocardiography may feel pressure or mechanical movement from the transducer as it is used to capture sound waves from the heart.

110
Q

What is the primary purpose of a multigated acquisition (MUGA) scan?

a. To evaluate coronary artery disease in asymptomatic patients

b. To measure the effect of various cardiac drugs on heart function

c. To assess heart valve abnormalities in elderly patients

d. To monitor the size of the abdominal aorta and detect aneurysms

A

b. To measure the effect of various cardiac drugs on heart function

Rationale: MUGA scans are commonly used to assess cardiac function, including the effects of cardiac or cardiotoxic drugs on heart performance.

111
Q

The nurse is assisting a patient undergoing nuclear imaging for myocardial perfusion. What is a key consideration during the procedure?

a. Ensuring the patient does not experience chest discomfort

b. Instructing the patient to refrain from using the restroom until the scan is complete

c. Asking the patient to lie with their arms extended overhead for 20 minutes

d. Giving the patient a meal immediately before the scan to aid in contrast absorption

A

c. Asking the patient to lie with their arms extended overhead for 20 minutes

Rationale: The patient must lie still, often with arms extended overhead, to minimize artifacts and ensure accurate imaging during nuclear perfusion imaging.

112
Q

Which of the following instructions should the nurse give to a patient scheduled for a nuclear imaging scan involving radioactive isotopes?

a. “You should not take any medications for 48 hours before the scan.”

b. “You should drink plenty of fluids immediately after the scan.”

c. “You will need to exercise for 30 minutes before the procedure.”

d. “You will need to stay still during the scan with your arms extended overhead.”

A

d. “You will need to stay still during the scan with your arms extended overhead.”

Rationale: The patient needs to stay still to ensure high-quality images and may be asked to extend their arms to minimize artifacts during the nuclear imaging scan.

113
Q

A patient scheduled for a positron emission tomography (PET) scan to evaluate myocardial perfusion asks why they need to check their blood glucose before the test. What is the best response by the nurse?

A. “Blood glucose levels must be within a specific range to ensure accurate results.”

B. “High blood glucose levels may cause the scan to be painful.”

C. “Glucose must be below 60 mg/dL to ensure the heart metabolizes the tracer correctly.”

D. “A high blood glucose level may indicate that you are not a good candidate for the test.”

A

A. “Blood glucose levels must be within a specific range to ensure accurate results.”

Rationale: PET scans use fluoro-18-deoxyglucose to assess myocardial metabolic activity, which competes with circulating glucose. If blood glucose levels are too high or too low, the uptake of the tracer may be altered, leading to inaccurate results. The optimal range is 60-140 mg/dL (3.3-7.8 μmol/L).

114
Q

A nurse is preparing a patient for a single-photon emission computed tomography (SPECT) scan. Which instruction is most important to provide?

A. “You will need to refrain from eating for at least 12 hours before the test.”

B. “You must avoid caffeine and tobacco for 24 hours before the scan.”

C. “You will need to lie completely still during the scan.”

D. “You should increase your fluid intake 12 hours before the scan.”

A

C. “You will need to lie completely still during the scan.”

Rationale: Movement can interfere with the imaging and affect the accuracy of results. While a short fasting period may be required, 12 hours is unnecessary. Caffeine and tobacco restrictions are more specific to stress tests rather than SPECT scans. Increased fluid intake is encouraged after the scan to help excrete the radioactive isotope.

115
Q

The nurse is reviewing a patient’s history before a transesophageal echocardiogram (TEE). Which finding requires immediate intervention?

A. The patient has a history of dysphagia.
B. The patient reports a sore throat from a recent cold.
C. The patient wears dentures.
D. The patient is mildly anxious about the procedure.

A

A. The patient has a history of dysphagia.

Rationale: A history of dysphagia raises concerns about the patient’s ability to swallow the probe safely, increasing the risk of aspiration. A sore throat is a minor concern, and dentures will be removed before the procedure. Mild anxiety is common and can be addressed with patient education and possibly sedation.

116
Q

Which statement by a patient scheduled for a stress echocardiogram indicates the need for further teaching?

A. “I will need to avoid caffeine for 12 hours before the test.”

B. “I can take a short break between exercise and the imaging.”

C. “The test will evaluate how my heart responds to stress.”

D. “I need to start exercising on a treadmill or bike before the images are taken.”

A

B. “I can take a short break between exercise and the imaging.”

Rationale: Post-exercise images must be taken within 1 minute to accurately compare wall motion before and after stress. Any delay could lead to misinterpretation of results.

117
Q

A nurse is monitoring a patient during a pharmacologic stress echocardiogram. Which symptom would require immediate intervention?

A. Mild nausea
B. Facial flushing
C. Severe dyspnea
D. Fatigue

A

C. Severe dyspnea

Rationale: Severe dyspnea may indicate bronchospasm or an adverse reaction to vasodilators such as regadenoson or dipyridamole. This requires immediate intervention, potentially with aminophylline to reverse the effects. Mild nausea, flushing, and fatigue are common and generally resolve without intervention.

118
Q

Which patient is contraindicated for a transesophageal echocardiogram (TEE)?

A. A patient with a history of severe GERD
B. A patient with a pacemaker
C. A patient on beta-blockers
D. A patient with an artificial heart valve

A

A. A patient with a history of severe GERD

Rationale: Severe GERD increases the risk of esophageal irritation, bleeding, or perforation during TEE. While pacemakers and artificial heart valves do not prevent a TEE, patients on beta-blockers may require adjustments due to their effect on heart rate during imaging.

119
Q

A nurse is reviewing post-procedure care for a patient who had a TEE. Which statement requires correction?

A. “You can eat and drink immediately after the test.”
B. “A sore throat is a common side effect.”
C. “You will need a driver if you were sedated.”
D. “We will monitor you until your sedation wears off.”

A

A. “You can eat and drink immediately after the test.”

Rationale: The gag reflex must return before the patient can eat or drink to prevent aspiration. A sore throat is expected, and sedation requires monitoring and a designated driver if done outpatient.

120
Q

A patient undergoing a PET scan asks why two different tracers are used. What is the best response?

A. “One tracer detects inflammation, and the other looks for clots.”

B. “Each tracer measures a different part of your heartbeat.”

C. “Using two tracers helps speed up the test results.”

D. “The first tracer checks for blood flow, and the second evaluates how your heart cells use glucose.”

A

D. “The first tracer checks for blood flow, and the second evaluates how your heart cells use glucose.”

Rationale: Nitrogen-13-ammonia assesses myocardial perfusion, while fluoro-18-deoxyglucose evaluates metabolic function. A mismatch indicates ischemia or damage.

121
Q

After a pharmacologic nuclear imaging test, which nursing intervention is most important?

A. Encourage fluid intake
B. Have the patient perform deep breathing exercises
C. Keep the patient NPO for 6 hours
D. Administer a bronchodilator immediately

A

A. Encourage fluid intake

Rationale: Fluids help flush out the radioactive tracer used in the test. Deep breathing is unnecessary, and NPO status is not required post-test. A bronchodilator is only needed if the patient experiences bronchospasm.

122
Q

A nurse is preparing a patient for a SPECT scan. Which statement by the patient requires immediate intervention?

A. “I am wearing my wedding ring.”
B. “I drank some coffee this morning.”
C. “I ate a light breakfast two hours ago.”
D. “I feel a little anxious about the test.”

A

B. “I drank some coffee this morning.”

Rationale: Caffeine should be avoided before certain nuclear tests, especially if combined with stress testing. Jewelry must be removed, but it does not require immediate intervention. A light meal is usually acceptable, and anxiety is common.

123
Q

Which patient requires close monitoring during a pharmacologic stress test?

A. A patient with a history of GERD
B. A patient with controlled hypertension
C. A patient with asthma
D. A patient with anemia

A

C. A patient with asthma

Rationale: Regadenoson and dipyridamole can trigger bronchospasm, making them dangerous for patients with asthma. Beta-agonists like aminophylline may be needed to reverse symptoms. GERD, hypertension, and anemia are not major concerns.

124
Q

A patient undergoing a PET scan asks how long the radioactive tracer will stay in their system. The nurse’s best response is:

A. “It will take several weeks to fully clear.”
B. “It should be gone within a few days.”
C. “Most of the tracer will lose its radioactivity within a few hours.”
D. “You will need to take a special medication to remove it.”

A

C. “Most of the tracer will lose its radioactivity within a few hours.”

Rationale: Most radioactive tracers used in PET scans decay quickly, usually within a few hours. Patients are advised to drink fluids to enhance elimination. No special medication is required.

125
Q

A 67-year-old male presents to the emergency department complaining of dizziness and palpitations. The nurse obtains a 12-lead ECG, which reveals an irregularly irregular rhythm with absent P waves and a narrow QRS complex. The patient’s blood pressure is 110/70 mmHg, and his heart rate is 140 bpm.

What is the most likely interpretation of this ECG finding?

A. First-degree atrioventricular (AV) block
B. Ventricular tachycardia
C. Sinus bradycardia
D. Atrial fibrillation

A

D. Atrial fibrillation

Rationale: The patient’s ECG findings are irregularly irregular rhythm, absent P waves, and a narrow QRS complex, which are hallmark features of atrial fibrillation (AFib). In AFib, chaotic atrial depolarizations replace normal P waves, and impulses reach the AV node irregularly, leading to an unpredictable ventricular response.

126
Q

A 45-year-old woman with a history of hypertension and fatigue underwent 24-hour Holter monitoring after experiencing occasional dizziness and palpitations. The results show frequent premature ventricular contractions (PVCs), occasional episodes of nonsustained ventricular tachycardia (NSVT), and an average heart rate of 85 bpm. The patient denies syncope.

Which intervention is most appropriate for the nurse to anticipate?

A. Immediate cardioversion
B. Initiating anticoagulation therapy
C. Educating the patient on lifestyle modifications and possible beta-blocker therapy
D. Preparing the patient for an implantable cardioverter-defibrillator (ICD)

A

C. Educating the patient on lifestyle modifications and possible beta-blocker therapy

Rationale: The patient has PVCs and NSVT but no symptoms of syncope or hemodynamic instability. In this case, lifestyle modifications (reducing caffeine, alcohol, and stress) and beta-blockers (e.g., metoprolol) are first-line treatments to reduce ventricular ectopy.

127
Q

A 58-year-old male with a history of diabetes and hyperlipidemia is scheduled for an exercise stress test to evaluate for coronary artery disease. He takes metformin, atorvastatin, and aspirin daily.

Which pre-test instructions should the nurse provide? (Select all that apply.)

A. “Avoid caffeine for 24 hours before the test.”

B. “Continue taking all of your medications as usual on the day of the test.”

C. “You should wear comfortable clothing and walking shoes.”

D. “Stop eating at least 4 hours before the test.”

E. “If you develop chest pain during the test, inform the technician immediately.”

A

A. “Avoid caffeine for 24 hours before the test.”
C. “You should wear comfortable clothing and walking shoes.”
D. “Stop eating at least 4 hours before the test.”
E. “If you develop chest pain during the test, inform the technician immediately.”

Rationale:
* Caffeine should be avoided for 24 hours as it can alter test results, particularly if vasodilators like adenosine or regadenoson are used in pharmacologic stress testing.

  • Comfortable clothing and walking shoes are essential since the patient will be walking or running on a treadmill.
  • Fasting for at least 4 hours before the test prevents nausea and reduces insulin-related fluctuations that could affect the ECG.
  • Reporting chest pain during the test is critical as it may indicate myocardial ischemia requiring test termination.
  • Metformin is often held the day of the test due to the risk of lactic acidosis if contrast dye is used in follow-up imaging.
128
Q

A 60-year-old male with a history of hypertension and smoking presents with severe chest pain radiating to his left arm. The emergency department nurse immediately obtains a 12-lead ECG, which shows ST-segment elevation in leads II, III, and aVF.

What is the priority nursing action?

A. Administer sublingual nitroglycerin and reassess
B. Obtain a troponin level and continue monitoring
C. Prepare the patient for emergent percutaneous coronary intervention (PCI)
D. Administer a beta-blocker to slow heart rate and decrease myocardial workload

A

C. Prepare the patient for emergent percutaneous coronary intervention (PCI)

Rationale: The ST-segment elevation in leads II, III, and aVF suggests an inferior wall myocardial infarction (MI), which is a medical emergency requiring immediate reperfusion therapy (PCI or thrombolysis).

129
Q

A 55-year-old male with chronic kidney disease arrives at the emergency department complaining of weakness and palpitations. His ECG reveals tall, peaked T waves, a widened QRS complex, and a prolonged PR interval.

Which intervention should the nurse anticipate?

A. Administering sodium bicarbonate
B. Preparing for emergent dialysis
C. Administering IV calcium gluconate
D. Giving atropine for bradycardia

A

C. Administering IV calcium gluconate

Rationale: The ECG findings of tall, peaked T waves, a widened QRS complex, and a prolonged PR interval are indicative of severe hyperkalemia, which can cause life-threatening arrhythmias.

130
Q

A 52-year-old female patient presents to the clinic with complaints of intermittent palpitations and dizziness that occur a few times a month. A 12-lead ECG performed in the clinic is normal, and a 24-hour Holter monitor previously did not capture any abnormalities. The provider recommends an event monitor.

What is the best explanation the nurse can provide to the patient regarding this test?

A. “You will wear this monitor for 2 to 4 weeks and press a button to record ECG activity when you feel symptoms.”

B. “An event monitor continuously records your heart rhythm for 24 hours, detecting any abnormal beats.”

C. “This device is implanted under your skin to provide long-term monitoring of your heart rhythm.”

D. “This test uses exercise and medication to induce an abnormal rhythm for closer evaluation.”

A

A. “You will wear this monitor for 2 to 4 weeks and press a button to record ECG activity when you feel symptoms.”

Rationale: An event monitor is ideal for capturing intermittent arrhythmias that do not occur daily. Unlike a Holter monitor, which records continuously for 24–48 hours, an event monitor allows patients to record ECG activity only when symptoms occur, increasing the likelihood of detecting abnormalities. Additionally, this test can be used for up to 4 weeks, making it superior for identifying infrequent dysrhythmias.

131
Q

A 70-year-old male with a history of atrial fibrillation and a dual-chamber pacemaker is scheduled for a routine pacemaker check via telephone transmission using an event monitor.

Which statement by the patient indicates a need for further teaching?

A. “This device can check my pacemaker function remotely.”
B. “I need to remain still while transmitting my heart rhythm over the phone.”
C. “If I feel lightheaded or dizzy, I should press the button to record my rhythm.”
D. “This test requires me to stop my blood thinners before sending my heart data.”

A

D. “This test requires me to stop my blood thinners before sending my heart data.”

Rationale: Event monitoring is a non-invasive procedure that does not require stopping anticoagulant therapy (e.g., warfarin, DOACs). Blood thinners are crucial for preventing stroke in atrial fibrillation patients and should never be stopped unless specifically instructed by a healthcare provider.

132
Q

A provider recommends an implantable loop recorder (ILR) for a 60-year-old patient with recurrent unexplained syncope that occurs every few months. The patient asks why an ILR is needed instead of a Holter monitor or event recorder.

What is the best response by the nurse?

A. “A Holter monitor would be a better option since it records continuously for 24 hours.”

B. “An ILR is implanted under your skin and continuously records your heart rhythm for months to years, making it ideal for capturing rare events.”

C. “An event monitor is superior because you can activate it when you feel symptoms.”

D. “You should have a treadmill stress test instead because it can provoke abnormal rhythms for better diagnosis.”

A

B. “An ILR is implanted under your skin and continuously records your heart rhythm for months to years, making it ideal for capturing rare events.”

Rationale: An ILR is the best option for patients with infrequent but potentially serious arrhythmias (e.g., unexplained syncope) because it provides long-term continuous monitoring (months to years). This increases the likelihood of detecting rare but significant rhythm disturbances that may not be captured by short-term monitoring methods.

133
Q

A nurse is educating a patient who will be wearing an external event monitor for 3 weeks. Which statements should be included in the teaching? (Select all that apply.)

A. “You must keep electrodes attached to your skin at all times.”

B. “Press the button when you feel symptoms so the device can record your ECG activity.”

C. “You should remove the monitor while sleeping to avoid interference.”

D. “Your ECG data can be transmitted to your healthcare provider in real-time or analyzed later.”

E. “If you experience severe chest pain, dizziness, or shortness of breath, seek emergency medical attention instead of just pressing the button.”

A

A. “You must keep electrodes attached to your skin at all times.”

B. “Press the button when you feel symptoms so the device can record your ECG activity.”

D. “Your ECG data can be transmitted to your healthcare provider in real-time or analyzed later.”

E. “If you experience severe chest pain, dizziness, or shortness of breath, seek emergency medical attention instead of just pressing the button.”

Rationale: The external event monitor requires continuous skin electrode attachment to ensure proper function. Patients must press the button when experiencing symptoms to capture the ECG rhythm at that moment. Many event monitors can transmit data in real time or store it for later analysis. In cases of severe symptoms, such as chest pain or shortness of breath, the patient should seek emergency care instead of relying solely on the monitor.

134
Q

A 40-year-old patient with a history of paroxysmal atrial fibrillation (AFib) is considering using a smartphone ECG application to monitor his heart rhythm at home. He asks how this technology works.
Which is the most appropriate response by the nurse?

A. “These apps use wrist-based sensors or finger contact to detect and record your heart rhythm, which you can share with your healthcare provider.”

B. “This device will provide continuous ECG monitoring and notify you of all arrhythmias immediately.”

C. “These apps can automatically diagnose heart rhythm disorders, so you won’t need to see a cardiologist.”

D. “A smartphone ECG application requires surgical implantation under the skin for continuous heart rhythm monitoring.”

A

A. “These apps use wrist-based sensors or finger contact to detect and record your heart rhythm, which you can share with your healthcare provider.”

Rationale: Smartphone ECG applications allow patients to check their heart rhythm by placing their fingers on a sensor or using a wrist-based device. The recorded data can then be saved or transmitted to a healthcare provider for evaluation. This technology is useful for tracking atrial fibrillation episodes and detecting irregular rhythms.

135
Q

A 58-year-old male with a history of hypertension and smoking reports chest discomfort that occurs only when climbing stairs or walking long distances. His resting ECG is normal, and his provider orders an exercise stress test.

What is the primary reason for performing this test?

A. To assess how the heart responds to increased oxygen demand during physical exertion

B. To evaluate heart function during a 6-minute period of walking on a flat surface

C. To provide continuous monitoring of heart rhythm for several weeks

D. To determine if a pacemaker is needed for the patient

A

A. To assess how the heart responds to increased oxygen demand during physical exertion

Rationale: An exercise stress test evaluates the heart’s response to physical stress by monitoring ECG and BP changes during exertion. This helps to detect coronary artery disease (CAD), exercise-induced ischemia, and arrhythmias that may not be evident at rest. This test is particularly useful in patients with symptoms that occur only with activity and can help determine exercise tolerance and safe activity levels.

136
Q

A 65-year-old female with a history of unstable angina and recent myocardial infarction (MI) is scheduled for an exercise stress test. Upon reviewing her history, the nurse notifies the provider to reassess the order.

Which aspect of the patient’s history requires further evaluation before proceeding with the test?

A. The patient has a history of hypertension controlled with medication
B. The patient had an MI 3 days ago and still experiences chest pain at rest
C. The patient walks with a cane due to mild arthritis
D. The patient has a pacemaker and previously underwent a Holter monitor test

A

B. The patient had an MI 3 days ago and still experiences chest pain at rest

Rationale: A recent myocardial infarction (within 3-5 days) and unstable angina (chest pain at rest) are contraindications to exercise stress testing. This test increases myocardial oxygen demand, which can provoke ischemia, worsening the patient’s condition. Patients with recent cardiac events should undergo further evaluation and stabilization before any stress testing.

137
Q

A nurse is preparing a patient for an exercise stress test. Which interventions should be included in the pre-procedure instructions? (Select all that apply.)

A. Instruct the patient to avoid caffeine and smoking for at least 24 hours before the test

B. Advise the patient to take all routine beta-blockers and nitrates on the morning of the test

C. Ensure the patient wears comfortable clothing and walking shoes for the test

D. Explain that ECG and blood pressure will be monitored throughout the exercise period

E. Inform the patient that chest pain or severe shortness of breath should be reported immediately during the test

A

A. Instruct the patient to avoid caffeine and smoking for at least 24 hours before the test
C. Ensure the patient wears comfortable clothing and walking shoes for the test
D. Explain that ECG and blood pressure will be monitored throughout the exercise period
E. Inform the patient that chest pain or severe shortness of breath should be reported immediately during the test

Rationale: Patients should avoid caffeine and smoking for at least 24 hours before the test, as these can affect heart rate and blood pressure. Comfortable clothing and proper footwear are important for safe exercise. Continuous ECG and BP monitoring are necessary to detect cardiac abnormalities. Patients must report symptoms like chest pain or shortness of breath immediately to prevent complications.

138
Q

A 72-year-old female with heart failure undergoes a 6-minute walk test to assess her functional status. She completes a distance of 250 meters but reports severe dyspnea and fatigue at the end of the test.
What does this result indicate?

A. The patient has a normal level of cardiac fitness
B. The patient has significant functional limitations and may require further intervention
C. The patient needs an implantable loop recorder to monitor heart activity
D. The test is inconclusive and should be repeated immediately

A

B. The patient has significant functional limitations and may require further intervention

Rationale: The 6-minute walk test assesses cardiac and respiratory endurance. A short walking distance with significant dyspnea or fatigue suggests poor functional capacity, which is common in heart failure and other cardiovascular conditions. These findings may prompt further evaluation and treatment adjustments.

139
Q

A nurse is educating a patient about an upcoming 6-minute walk test. The patient asks, “Why am I doing this test?”

Which response by the nurse is most appropriate?

A. “This test is used to determine how well your heart responds to high-intensity exercise.”

B. “This test will detect abnormal heart rhythms that occur only during physical activity.”

C. “This test will determine if you need a pacemaker for abnormal heart function.”

D. “This test evaluates your baseline functional capacity and can monitor your response to treatment over time.”

A

D. “This test evaluates your baseline functional capacity and can monitor your response to treatment over time.”

Rationale: The 6-minute walk test is a general measure of cardiac and pulmonary fitness that helps assess a patient’s exercise tolerance and response to treatment over time. It is commonly used in older adults, heart failure patients, and those undergoing rehabilitation.

140
Q

A 55-year-old male is undergoing an exercise stress test on a treadmill. During the test, his ECG shows ST-segment depression, and he reports chest tightness and dizziness.

What is the priority nursing action?

A. Instruct the patient to slow down and continue walking
B. Administer a beta-blocker to reduce heart rate and allow the test to continue
C. Immediately stop the test and place the patient in a supine position
D. Encourage the patient to take slow, deep breaths and complete the test

A

C. Immediately stop the test and place the patient in a supine position

Rationale: ST-segment depression and chest pain indicate myocardial ischemia, which can progress to a heart attack. The test must be stopped immediately to prevent further cardiac stress. Placing the patient in a supine position helps improve circulation, and further interventions (e.g., oxygen, nitroglycerin, ECG monitoring) may be needed.

141
Q

A 68-year-old male with a history of hypertension and heart failure presents with increasing shortness of breath and lower extremity swelling. A chest x-ray is ordered to evaluate his symptoms. The radiology report indicates cardiomegaly and pulmonary congestion.

Based on these findings, which condition is the most likely cause of the patient’s symptoms?

A. Chronic obstructive pulmonary disease (COPD)
B. Aortic aneurysm
C. Heart failure
D. Pericarditis

A

C. Heart failure

Rationale: A chest x-ray showing cardiomegaly and pulmonary congestion is highly suggestive of heart failure. Cardiomegaly indicates an enlarged heart, often due to chronic volume overload or myocardial dysfunction. Pulmonary congestion results from fluid backing up into the lungs, leading to dyspnea, crackles, and lower extremity edema. These findings are characteristic of heart failure, particularly left-sided heart failure.

142
Q

A nurse is educating a patient who is scheduled for a chest x-ray due to suspected cardiac disease. The patient asks, “What can this test show about my heart?”

Which response by the nurse is most appropriate?

A. “A chest x-ray provides a detailed image of your coronary arteries to detect blockages.”

B. “A chest x-ray can show if your heart is enlarged or if there is fluid buildup around it.”

C. “A chest x-ray measures how well your heart pumps blood to the rest of your body.”

D. “A chest x-ray is primarily used to detect electrical abnormalities in your heart rhythm.”

A

B. “A chest x-ray can show if your heart is enlarged or if there is fluid buildup around it.”

Rationale: A chest x-ray assesses the size, shape, and position of the heart and can identify conditions such as cardiomegaly, pericardial effusion, and pulmonary congestion. However, it does not provide detailed images of coronary arteries, measure cardiac output, or detect arrhythmias. Those require specialized tests like coronary angiography, echocardiography, or electrocardiography (ECG).

143
Q

A nurse is preparing a patient for a chest x-ray to evaluate for cardiac abnormalities. Which interventions should the nurse perform before the procedure? (Select all that apply.)

A. Instruct the patient to remove any metallic objects or jewelry from the chest and neck area

B. Inform the patient that the procedure is noninvasive and painless

C. Administer an IV contrast agent to enhance visualization of the heart

D. Verify that the patient is not pregnant before proceeding with the x-ray

E. Explain that the patient must hold their breath while the x-ray is taken

A

A. Instruct the patient to remove any metallic objects or jewelry from the chest and neck area

B. Inform the patient that the procedure is noninvasive and painless

D. Verify that the patient is not pregnant before proceeding with the x-ray

E. Explain that the patient must hold their breath while the x-ray is taken

Rationale: Before a chest x-ray, the nurse should ensure that the patient removes all metal objects to prevent interference with imaging, confirm that the patient is not pregnant, and explain that breath-holding is required for clear images. The procedure is noninvasive and painless. IV contrast is not required for a routine chest x-ray but is used for more advanced imaging, such as CT scans.

144
Q

A 72-year-old female with a history of rheumatic fever and exertional dyspnea is referred for an echocardiogram to assess for possible valvular disease. The echocardiogram reveals thickened mitral valve leaflets with restricted motion and decreased ejection fraction.

Which condition is most likely causing the patient’s symptoms?

A. Mitral stenosis
B. Aortic stenosis
C. Tricuspid regurgitation
D. Pulmonary hypertension

A

A. Mitral stenosis

Rationale: Mitral stenosis is characterized by thickened, immobile mitral valve leaflets leading to impaired blood flow from the left atrium to the left ventricle. This causes exertional dyspnea, fatigue, and reduced cardiac output. Rheumatic fever is a common cause of mitral stenosis, and echocardiography is the gold standard for diagnosis.

145
Q

A patient undergoing an echocardiogram is found to have an ejection fraction (EF) of 35%. The nurse understands that this finding is most consistent with:

A. Normal left ventricular function
B. Atrial fibrillation
C. Pulmonary embolism
D. Systolic heart failure

A

D. Systolic heart failure

Rationale: Ejection fraction measures the percentage of blood ejected from the left ventricle during systole. A normal EF is 55-70%. An EF of 35% indicates systolic heart failure, where the ventricle is unable to pump effectively, leading to reduced cardiac output and fluid overload.

146
Q

A nurse is preparing a patient for a transthoracic echocardiogram (TTE). Which interventions should the nurse perform? (Select all that apply.)

A. Explain that the procedure is noninvasive and painless

B. Ensure the patient removes any metal jewelry from the chest area

C. Keep the patient NPO for at least 6 hours before the procedure

D. Inform the patient that sedation is not required

E. Position the patient in a left lateral position during the test

A

A. Explain that the procedure is noninvasive and painless
D. Inform the patient that sedation is not required
E. Position the patient in a left lateral position during the test

Rationale: A **TTE is noninvasive, does not require sedation, and is performed with the patient in the left lateral position to improve imaging. Unlike a TEE, fasting is not required since no sedation or esophageal intubation is involved.

147
Q

A patient with atrial fibrillation is scheduled for a TEE to evaluate for left atrial thrombus before cardioversion. The nurse reviews the patient’s history and identifies which finding as a contraindication to the procedure?

A. Hypertension
B. Esophageal stricture
C. Left ventricular hypertrophy
D. Peripheral arterial disease

A

B. Esophageal stricture

Rationale: TEE requires passing an endoscope through the esophagus, so conditions like esophageal stricture, dysphagia, or prior radiation therapy to the chest can increase the risk of esophageal perforation.

148
Q

A 55-year-old male with diabetes and exertional chest pain undergoes a stress echocardiogram. After exercise, his echocardiogram shows hypokinesis (reduced wall motion) in the left anterior descending artery (LAD) territory.

What does this result indicate?

A. Normal myocardial function
B. Coronary artery disease (CAD) affecting the LAD
C. Hypertrophic cardiomyopathy
D. Pulmonary hypertension

A

B. Coronary artery disease (CAD) affecting the LAD

Rationale: Stress echocardiography assesses myocardial function during increased oxygen demand. Hypokinesis in a specific region suggests ischemia due to CAD, particularly in the LAD territory.

149
Q

A bubble study is performed during an echocardiogram to evaluate for:

A. Coronary artery disease
B. Aortic stenosis
C. Patent foramen ovale (PFO)
D. Pericardial effusion

A

C. Patent foramen ovale (PFO)

Rationale: A bubble study detects right-to-left shunting, which is seen in PFO or atrial septal defects.

150
Q

The nurse is monitoring a patient after a TEE. Which complications should the nurse assess for? (Select all that apply.)

A. Esophageal perforation
B. Dysrhythmias
C. Stroke
D. Pneumothorax
E. Vasovagal reaction

A

A. Esophageal perforation
B. Dysrhythmias
C. Stroke
E. Vasovagal reaction

Rationale: TEE complications include esophageal trauma, dysrhythmias, stroke (due to embolization), and vasovagal reactions. Pneumothorax is not a risk since the lungs are not directly involved.

151
Q

A patient unable to perform exercise stress testing is given dobutamine for a pharmacologic stress echocardiogram.

What is the primary action of dobutamine in this setting?

A. Slows heart rate to improve image clarity
B. Induces controlled ischemia in the coronary arteries
C. Increases heart rate and myocardial oxygen demand
D. Dilates coronary arteries to enhance perfusion

A

C. Increases heart rate and myocardial oxygen demand

Rationale: Dobutamine is a beta-agonist that increases heart rate and myocardial contractility, simulating exercise. This helps assess for ischemic changes on echocardiography.

152
Q

Which echocardiographic technique provides the most precise images for diagnosing mitral valve disease?

A. Transesophageal echocardiogram (TEE)
B. Transthoracic echocardiogram (TTE)
C. Doppler echocardiography
D. Stress echocardiography

A

A. Transesophageal echocardiogram (TEE)

Rationale: TEE provides better visualization of posterior heart structures, including the mitral valve, without interference from the chest wall.

153
Q

A patient’s echocardiogram with color Doppler imaging reveals red and blue flow in the same region of the mitral valve. This finding suggests:

A. Valvular regurgitation
B. Normal valve function
C. Pericardial effusion
D. Myocardial infarction

A

A. Valvular regurgitation

Rationale: Color Doppler identifies abnormal blood flow patterns, and mixed colors suggest turbulent regurgitant flow.

154
Q

Which conditions may require an echocardiogram for diagnosis? (Select all that apply.)

A. Heart failure
B. Atrial fibrillation
C. Myocardial infarction
D. Deep vein thrombosis
E. Valvular disease

A

A. Heart failure
B. Atrial fibrillation
C. Myocardial infarction
E. Valvular disease

Rationale: Echocardiography is used for heart failure, atrial fibrillation (to check for clots), MI (to assess wall motion), and valvular disease. It is not used for deep vein thrombosis.

155
Q

A patient with atrial fibrillation is being evaluated for left atrial thrombus before undergoing cardioversion. Which echocardiographic technique is the most appropriate for detecting an atrial thrombus?

A. Transthoracic echocardiogram (TTE)
B. Transesophageal echocardiogram (TEE)
C. Stress echocardiography
D. M-mode echocardiography

A

B. Transesophageal echocardiogram (TEE)

Rationale: TEE provides better visualization of the left atrium compared to TTE since the esophagus is closer to the posterior heart structures. Left atrial thrombus is a concern in atrial fibrillation and must be ruled out before cardioversion to prevent embolization.

156
Q

A nurse is reviewing a patient’s echocardiogram results, which included color Doppler imaging. This technique is most useful for diagnosing which conditions? (Select all that apply.)

A. Mitral regurgitation
B. Aortic stenosis
C. Left ventricular hypertrophy
D. Tricuspid regurgitation
E. Atrial septal defect

A

A. Mitral regurgitation
B. Aortic stenosis
D. Tricuspid regurgitation
E. Atrial septal defect

Rationale: Color Doppler visualizes the speed and direction of blood flow, making it useful for identifying valvular disorders (mitral and tricuspid regurgitation, aortic stenosis) and congenital defects (atrial septal defect). Left ventricular hypertrophy is best evaluated by 2D echocardiography.

157
Q

Which statement accurately describes the difference between M-mode echocardiography and 2D echocardiography?

A. M-mode provides real-time 3D images of the heart

B. M-mode provides a single-dimensional view, while 2D echocardiography provides cross-sectional views

C. 2D echocardiography only measures chamber size, while M-mode assesses wall motion

D. M-mode is preferred for diagnosing coronary artery disease

A

B. M-mode provides a single-dimensional view, while 2D echocardiography provides cross-sectional views

Rationale: M-mode records intracardiac structure motion along a single US beam, measuring wall thickness and chamber size. 2D echocardiography provides a cross-sectional view of heart structures to assess spatial relationships.

158
Q

A patient with limited mobility undergoes a pharmacologic stress echocardiogram with dipyridamole. What is the expected physiological response in a healthy patient?

A. Vasodilation of normal coronary arteries
B. Decreased heart rate and contractility
C. Coronary artery spasm
D. Increased myocardial oxygen demand

A

A. Vasodilation of normal coronary arteries

Rationale: Dipyridamole is a vasodilator that increases coronary blood flow in healthy arteries. Diseased arteries do not dilate as well, allowing for detection of ischemia on echocardiography.

159
Q

A nurse is providing pre-procedure instructions for a patient scheduled for a transesophageal echocardiogram (TEE). Which statements should be included in the teaching? (Select all that apply.)

A. “You will need to remain NPO for at least 6 hours before the procedure.”

B. “Sedation will be used, so you will need someone to drive you home.”

C. “You may experience a sore throat after the procedure.”

D. “You must lie completely flat during the procedure.”

E. “This procedure is completely noninvasive.”

A

A. “You will need to remain NPO for at least 6 hours before the procedure.”
B. “Sedation will be used, so you will need someone to drive you home.”
C. “You may experience a sore throat after the procedure.”

Rationale: TEE requires NPO status due to sedation and esophageal intubation. A sore throat is common post-procedure. The patient should be positioned in a left lateral position, not completely flat. Unlike a TTE, TEE is semi-invasive.

160
Q

A patient with chest pain is scheduled for a coronary CT angiography (CTA). What is the primary advantage of this test compared to cardiac catheterization?

A. It is more effective at identifying coronary artery blockages
B. It is a noninvasive test with less radiation exposure
C. It allows for immediate angioplasty if needed
D. It is the gold standard for diagnosing coronary artery disease (CAD)

A

B. It is a noninvasive test with less radiation exposure

Rationale: Coronary CTA is noninvasive, requires less radiation than cardiac catheterization, and is useful for visualizing coronary circulation. However, cardiac catheterization remains the gold standard for diagnosing CAD and allows for intervention if blockages are found.

161
Q

A nurse is preparing a patient for a coronary CT angiography (CTA). Which factors must be considered before the test? (Select all that apply.)

A. The patient must be in normal sinus rhythm
B. IV contrast may be used, requiring an allergy assessment
C. The test is invasive and requires a recovery period
D. The patient should avoid caffeine before the test
E. The test can be performed on pregnant women without concern

A

A. The patient must be in normal sinus rhythm
B. IV contrast may be used, requiring an allergy assessment
D. The patient should avoid caffeine before the test

Rationale: Coronary CTA requires normal sinus rhythm for accurate imaging. IV contrast is often used, so an allergy assessment is necessary. Caffeine should be avoided before the test to prevent increased heart rate, which can interfere with imaging. The test is noninvasive and is not recommended for pregnant women due to radiation exposure.

162
Q

A calcium-scoring CT scan is primarily used for which purpose?

A. Detecting early coronary artery calcification
B. Measuring myocardial perfusion
C. Identifying structural heart defects
D. Assessing heart valve function

A

A. Detecting early coronary artery calcification

Rationale: A calcium-scoring CT scan detects calcium deposits in coronary plaque before symptoms appear, helping to predict the risk of future cardiac events. It does not measure myocardial perfusion, structural heart defects, or valve function.

163
Q

Which statement best describes the function of electron beam CT (EBCT)?

A. It measures blood flow through the coronary arteries
B. It detects coronary calcium deposits in plaques
C. It identifies areas of myocardial infarction
D. It evaluates heart valve motion

A

B. It detects coronary calcium deposits in plaques

Rationale: EBCT is the most common method used for calcium-scoring CT scans, detecting coronary calcium deposits and helping assess the risk of coronary artery disease (CAD).

164
Q

A nurse is reviewing a patient’s history before a cardiac CT scan. Which factors can interfere with image accuracy? (Select all that apply.)

A. Irregular heart rhythms
B. Excessive movement during the scan
C. Presence of a pacemaker
D. Use of IV contrast
E. Tachycardia

A

A. Irregular heart rhythms
B. Excessive movement during the scan
C. Presence of a pacemaker
E. Tachycardia

Rationale: Irregular heart rhythms (Atrial fibrillation, PVCs) can distort images. Patient movement can reduce scan clarity. A pacemaker can cause artifacts that interfere with image quality. Tachycardia affects scan accuracy because higher heart rates make it harder to capture clear images. IV contrast (D) does not interfere with accuracy but is required for some CT scans.

165
Q

A patient is suspected to have a severe coronary blockage and may need immediate stent placement. Which diagnostic test is most appropriate?

A. Coronary CT angiography (CTA)
B. Calcium-scoring CT scan
C. Cardiac catheterization
D. Electron beam CT (EBCT)

A

C. Cardiac catheterization

Rationale: Cardiac catheterization is the gold standard for diagnosing coronary artery disease (CAD) and allows for immediate intervention (angioplasty or stent placement) if blockages are found.

166
Q

A nurse is assessing a patient scheduled for a coronary CTA with IV contrast. Which assessment finding requires immediate intervention?

A. History of iodine allergy
B. Heart rate of 70 bpm
C. Previous cardiac catheterization
D. Mild hypertension (140/90 mmHg)

A

A. History of iodine allergy

Rationale: IV contrast used in cardiac CT scans contains iodine, which can cause severe allergic reactions in sensitive individuals. Pre-medication with steroids or antihistamines may be needed, or an alternative imaging test should be considered.

167
Q

A nurse is educating a patient about coronary CT angiography (CTA). Which statements about potential risks and limitations should be included? (Select all that apply.)

A. The test involves radiation exposure
B. IV contrast dye may cause allergic reactions
C. The test can substitute for a cardiac catheterization
D. It may not be suitable for patients with arrhythmias
E. It can detect coronary artery calcium deposits

A

A. The test involves radiation exposure
B. IV contrast dye may cause allergic reactions
D. It may not be suitable for patients with arrhythmias
E. It can detect coronary artery calcium deposits

Rationale: Coronary CTA exposes the patient to radiation, requires IV contrast, and is less effective in patients with arrhythmias due to motion artifacts. It cannot fully replace cardiac catheterization, as intervention (angioplasty/stenting) cannot be performed with CTA alone. It can detect calcium deposits in coronary arteries.

168
Q

A 55-year-old male with a family history of coronary artery disease (CAD) wants to assess his risk of developing heart disease but has no symptoms. Which test is most appropriate?

A. Stress echocardiography
B. Coronary angiography
C. Calcium-scoring CT scan
D. Transesophageal echocardiography

A

C. Calcium-scoring CT scan

Rationale: A calcium-scoring CT scan is noninvasive and helps detect coronary artery calcium deposits, which can indicate early coronary artery disease (CAD) before symptoms develop. It is useful for risk assessment in asymptomatic patients with a strong family history.

169
Q

Which is the primary advantage of cardiovascular magnetic resonance imaging (CMRI) over other imaging techniques?

A. It provides radiation-free imaging
B. It is more cost-effective than other imaging modalities
C. It provides the ability to visualize coronary stents
D. It is the gold standard for diagnosing myocardial infarction (MI)

A

A. It provides radiation-free imaging

Rationale: One of the main advantages of CMRI is that it does not involve radiation, making it a safer option for patients who require multiple imaging tests, especially when monitoring heart function over time.

170
Q

A nurse is educating a patient about cardiovascular magnetic resonance imaging (CMRI). Which conditions can CMRI help diagnose or assess? (Select all that apply.)

A. Myocardial infarction (MI)
B. Congenital heart disorders
C. Coronary artery disease (CAD)
D. Aortic disorders
E. Ventricular arrhythmias

A

A. Myocardial infarction (MI)
B. Congenital heart disorders
C. Coronary artery disease (CAD)
D. Aortic disorders

Rationale: CMRI is useful for diagnosing myocardial infarction (MI), congenital heart disorders, coronary artery disease (CAD), and aortic disorders. It is not typically used to diagnose ventricular arrhythmias, as other diagnostic tools like EKGs are more effective for this purpose.

171
Q

A patient with a pacemaker is scheduled for cardiovascular magnetic resonance imaging (CMRI). What is the most important consideration?

A. The patient must have a new model pacemaker that is MRI-compatible

B. The patient should avoid eating for 24 hours before the test

C. The pacemaker will be removed before the procedure

D. CMRI can be performed immediately after pacemaker placement

A

A. The patient must have a new model pacemaker that is MRI-compatible

Rationale: Newer models of pacemakers are MRI-compatible, but older models may be affected by the magnetic fields, potentially altering their function. CMRI should not be performed immediately after pacemaker placement, and patients are not required to fast.

172
Q

Which imaging test is most sensitive in detecting small myocardial infarctions (MIs) that might be missed by other imaging methods?

A. Single-photon emission computed tomography (SPECT)
B. Cardiovascular magnetic resonance imaging (CMRI)
C. Coronary computed tomography angiography (CTA)
D. Echocardiography

A

B. Cardiovascular magnetic resonance imaging (CMRI)

Rationale: CMRI is highly sensitive in detecting even small myocardial infarctions (MIs), providing three-dimensional imaging that can reveal subtle changes in myocardial tissue that might not be visible with SPECT or other tests.

173
Q

A patient who underwent coronary stent placement 8 weeks ago is scheduled for a cardiovascular magnetic resonance imaging (CMRI). What is the nurse’s best response?

A. CMRI can be performed immediately as long as the patient is stable
B. CMRI can be performed as long as the patient does not have a pacemaker
C. CMRI should be postponed until 6 weeks after stent placement
D. CMRI is contraindicated for this patient because of the coronary stent

A

C. CMRI should be postponed until 6 weeks after stent placement

Rationale: Patients with coronary stents are generally advised to wait 6 weeks after the procedure before undergoing CMRI to ensure the stent is securely in place and to minimize the risk of dislodging the stent during imaging.

174
Q

Why is cardiovascular magnetic resonance imaging (CMRI) discouraged for patients with older model pacemakers or implantable cardioverter defibrillators (ICDs)?

A. The device will be damaged by the magnetic field
B. The test will cause pain for the patient
C. CMRI is ineffective in patients with these devices
D. The MRI may interfere with the device’s function

A

D. The MRI may interfere with the device’s function

Rationale: The strong magnetic field of CMRI can interfere with the function of older model pacemakers and ICDs, potentially altering their settings or causing malfunctions. However, newer models of pacemakers and ICDs are MRI-compatible and can be safely used during CMRI.

175
Q

Which aspect of coronary artery disease (CAD) can cardiovascular magnetic resonance imaging (CMRI) help assess?

A. Coronary plaque composition
B. Coronary artery stenosis
C. Aneurysm formation in the coronary arteries
D. The extent of ischemic injury to the myocardium

A

D. The extent of ischemic injury to the myocardium

CMRI is particularly useful for assessing myocardial damage and ischemic injury caused by coronary artery disease (CAD). While it does not directly assess coronary stenosis or plaque composition, it can evaluate the effects of CAD, such as areas of ischemic injury and heart function post-infarction.

176
Q

What is the primary use of the multigated acquisition (MUGA) or cardiac blood pool scan in nuclear cardiology?

A. To assess ventricular wall motion and ejection fraction (EF)
B. To detect coronary artery disease (CAD)
C. To assess the viability of myocardial tissue
D. To evaluate the success of coronary artery bypass grafting (CABG)

A

A. To assess ventricular wall motion and ejection fraction (EF)

Rationale: The MUGA scan primarily provides information about ventricular wall motion during systole and diastole and is used to assess ejection fraction (EF), an important indicator of heart function.

177
Q

What is a primary indication for stress perfusion imaging in a patient with suspected coronary artery disease (CAD)?

A. To evaluate cardiac valve function
B. To determine the risk of myocardial infarction
C. To assess coronary blood flow changes with exercise or pharmacologic stress
D. To measure blood pressure during physical exertion

A

C. To assess coronary blood flow changes with exercise or pharmacologic stress

Rationale: Stress perfusion imaging is used to assess changes in coronary blood flow with exercise or pharmacological agents, helping to diagnose CAD and determine the severity of ischemia or blood flow abnormalities.

178
Q

Which of the following conditions can be diagnosed or assessed using stress perfusion imaging? (Select all that apply.)

A. Coronary artery disease (CAD)
B. Viability of myocardial tissue
C. Heart valve abnormalities
D. Success of coronary artery bypass surgery (CABG)
E. Left ventricular hypertrophy

A

A. Coronary artery disease (CAD)
B. Viability of myocardial tissue
D. Success of coronary artery bypass surgery (CABG)

Rationale: Stress perfusion imaging is used to assess CAD, determine myocardial tissue viability, and evaluate the success of interventions like coronary artery bypass surgery (CABG). It is not typically used to diagnose heart valve abnormalities or left ventricular hypertrophy.

179
Q

What is the primary advantage of positron emission tomography (PET) in myocardial perfusion imaging?

A. Lower cost compared to other imaging modalities
B. Requires no pharmacologic stress agents
C. Higher radiation exposure to the patient
D. Higher sensitivity for detecting myocardial ischemia and viability

A

D. Higher sensitivity for detecting myocardial ischemia and viability

Rationale: PET offers high sensitivity for detecting myocardial ischemia and assessing the viability of myocardial tissue, making it an increasingly popular choice for evaluating CAD and other heart conditions.

180
Q

When is exercise stress perfusion imaging preferred over pharmacologic stress imaging in patients?

A. When the patient has no contraindications to physical exercise
B. When the patient is unable to tolerate exercise
C. When the patient has a history of severe CAD
D. When the patient needs to avoid radiation exposure

A

A. When the patient has no contraindications to physical exercise

Rationale: Exercise stress perfusion imaging is preferred when the patient is physically able to exercise and has no contraindications. It provides the most accurate assessment of coronary blood flow and ischemia under physical stress.

181
Q

Which of the following is a common imaging agent used in single-photon emission computed tomography (SPECT) for coronary artery disease (CAD) diagnosis?

A. Rubidium-82
B. Technetium, Thallium, or Sestamibi
C. Regadenoson
D. Adenosine

A

B. Technetium, Thallium, or Sestamibi

Rationale: Common SPECT imaging agents include Technetium, Thallium, and Sestamibi, which help assess myocardial perfusion and ischemia in CAD patients. Rubidium-82 is used in PET scans, and regadenoson and adenosine are pharmacologic agents used for stress testing.

182
Q

Which condition can lead to false-positive results in SPECT imaging?

A. Obesity
B. Aortic stenosis
C. Atrial fibrillation
D. Coronary artery bypass surgery

A

A. Obesity

Rationale: Obesity can lead to false-positive results in SPECT imaging due to difficulty in obtaining clear images, which can cause misinterpretation of coronary perfusion. Other factors, such as multivessel coronary disease, may lead to false-negative results.

183
Q

Why would IV adenosine be used in a stress perfusion imaging test when a patient is unable to exercise?

A. To increase heart rate and simulate physical exercise
B. To reduce myocardial oxygen demand during the test
C. To improve image quality by enhancing myocardial perfusion
D. To dilate coronary arteries and simulate the effects of exercise

A

D. To dilate coronary arteries and simulate the effects of exercise

Rationale: IV adenosine is used to dilate coronary arteries, simulating the effects of exercise for patients who cannot perform physical activity. This allows for stress testing and evaluation of myocardial perfusion and ischemia.

184
Q

What is the primary purpose of cardiac catheterization?

A. To assess intracardiac pressures, oxygen levels, and ejection fraction (EF)
B. To treat atrial fibrillation
C. To provide electrocardiographic monitoring
D. To measure serum cholesterol levels

A

A. To assess intracardiac pressures, oxygen levels, and ejection fraction (EF)

Rationale: Cardiac catheterization is primarily used to measure intracardiac pressures, oxygen levels, cardiac output (CO), and ejection fraction (EF), as well as to visualize the coronary arteries and heart chambers.

185
Q

What is coronary angiography primarily used for during a left-sided heart catheterization?

A. To evaluate ventricular wall motion
B. To measure pulmonary capillary wedge pressure (PAWP)
C. To visualize the coronary arteries and detect blockages
D. To assess the ejection fraction (EF)

A

C. To visualize the coronary arteries and detect blockages

Rationale: Coronary angiography, performed during left-sided heart catheterization, is used to visualize the coronary arteries and identify any blockages, allowing for the assessment of coronary artery disease (CAD).

186
Q

Which of the following are potential complications of cardiac catheterization? (Select all that apply.)

A. Bleeding or hematoma at the puncture site
B. Infection at the catheter insertion site
C. Electrolyte imbalances
D. Thrombus formation
E. Stroke

A

A. Bleeding or hematoma at the puncture site
B. Infection at the catheter insertion site
D. Thrombus formation
E. Stroke

Rationale: Common complications of cardiac catheterization include bleeding or hematoma at the puncture site, infection, thrombus formation, and stroke. Electrolyte imbalances are not typically considered a direct complication of the procedure.

187
Q

Where can the catheter be inserted for a right-sided cardiac catheterization?

A. Femoral, internal jugular, subclavian, or antecubital vein
B. Radial, femoral, or brachial artery
C. Coronary artery
D. Superior vena cava, internal jugular, subclavian, or antecubital vein

A

A. Femoral, internal jugular, subclavian, or antecubital vein

Rationale: For right-sided cardiac catheterization, the catheter is inserted through veins, such as the femoral, internal jugular, subclavian, or antecubital veins. This allows the catheter to access the right side of the heart.

188
Q

What sensation do patients often experience when contrast media is injected during cardiac catheterization?

A. Coldness in the legs
B. Flushed sensation
C. Palpitations
D. Shortness of breath

A

B. Flushed sensation

Rationale: Patients typically report a temporary flushed sensation when the contrast media is injected during cardiac catheterization. This is a common, non-harmful reaction.

189
Q

During a left-sided heart catheterization, where is the catheter inserted to reach the left ventricle?

A. Radial femoral or brachial artery
B. Right atrium or brachial artery
C. Femoral vein or brachial artery
D. Coronary artery or brachial artery

A

A. Radial femoral or brachial artery

Rationale: For left-sided heart catheterization, the catheter is typically inserted into the radial, femoral, or brachial artery and then advanced retrogradely up to the left ventricle.

190
Q

What is the purpose of measuring pulmonary capillary wedge pressure (PAWP) during right-sided cardiac catheterization?

A. To determine the oxygen saturation of the blood
B. To monitor cardiac output
C. To assess the left-sided heart function
D. To visualize the coronary arteries

A

C. To assess the left-sided heart function

Rationale: PAWP is measured during right-sided cardiac catheterization to estimate left-sided heart function, particularly the left atrial pressure, which provides insight into the function of the left side of the heart.

191
Q

Which of the following is a common catheter insertion site for left-sided cardiac catheterization?

A. Radial artery
B. Femoral vein
C. Internal jugular vein
D. Femoral artery

A

D. Femoral artery

Rationale: A common insertion site for left-sided cardiac catheterization is the femoral artery, through which the catheter is passed retrograde to the left side of the heart.

192
Q

Which of the following is essential to monitor during cardiac catheterization to detect potential complications?

A. Heart rate and blood pressure
B. Body temperature
C. Bowel sounds
D. Urine output

A

A. Heart rate and blood pressure

Rationale: Heart rate and blood pressure must be closely monitored during cardiac catheterization to detect potential complications, such as dysrhythmias, bleeding, or hypotension.

193
Q

What is a potential risk when a catheter is kinked or looped during cardiac catheterization?

A. Increased radiation exposure
B. Damage to coronary arteries
C. Incorrect measurement of blood pressure
D. Faster contrast media injection

A

C. Incorrect measurement of blood pressure

Rationale: If the catheter becomes kinked or looped, it may result in incorrect measurement of intracardiac pressures, which can affect the diagnosis and management of cardiac conditions.

194
Q

What is the primary purpose of intravascular ultrasound (IVUS)?

A. To evaluate heart valve function
B. To assess the composition, location, and thickness of plaque in the coronary arteries
C. To monitor electrical activity in the heart
D. To measure ejection fraction (EF)

A

B. To assess the composition, location, and thickness of plaque in the coronary arteries

Rationale: IVUS is primarily used to evaluate the composition, location, and thickness of plaque within the coronary arteries, providing detailed information on the health of the arterial walls.

195
Q

What type of imaging does intravascular ultrasound (IVUS) provide?

A. 3D or 2D images
B. X-ray images
C. CT scan images
D. MRI images

A

A. 3D or 2D images

Rationale: IVUS provides 2D or 3D images of the arterial walls of the coronary arteries, offering a detailed cross-sectional view of the blood vessels.

196
Q

How is intravascular ultrasound (IVUS) typically performed?

A. By inserting a small catheter with a miniature transducer to obtain ultrasound images from within the artery

B. By inserting a catheter through the femoral vein to access the right side of the heart

C. By using a needle biopsy to obtain tissue samples from the coronary artery

D. By injecting contrast dye to visualize the coronary arteries

A

A. By inserting a small catheter with a miniature transducer to obtain ultrasound images from within the artery

Rationale: IVUS involves inserting a small catheter with a miniature transducer into the artery to obtain ultrasound images of the arterial walls, which helps assess plaque composition and arterial health.

197
Q

Intravascular ultrasound (IVUS) is often performed in addition to which of the following procedures?

A. Cardiac MRI
B. Coronary angiography
C. Electrocardiogram (ECG)
D. Electrophysiology study

A

B. Coronary angiography

Rationale: IVUS is commonly performed alongside coronary angiography to provide detailed information about plaque and arterial wall health during coronary interventions.

198
Q

Which of the following is the primary focus of nursing care after intravascular ultrasound (IVUS)?

A. Monitoring for electrical disturbances
B. Ensuring the patient remains hydrated post-procedure
C. Performing routine exercise stress testing
D. Monitoring for bleeding or hematoma at the catheter insertion site

A

D. Monitoring for bleeding or hematoma at the catheter insertion site

Rationale: Post-procedure care after IVUS is similar to care after cardiac catheterization, with a focus on monitoring for bleeding or hematoma at the catheter insertion site and other potential complications.

199
Q

Which of the following can intravascular ultrasound (IVUS) evaluate?

A. Vessel response to treatments such as stent placement and atherectomy
B. Heart rhythm abnormalities
C. Valve function in the coronary arteries
D. Coronary artery bypass graft (CABG) patency

A

A. Vessel response to treatments such as stent placement and atherectomy

Rationale: IVUS is useful for evaluating how the coronary arteries respond to treatments such as stent placement and atherectomy, providing detailed feedback on the effectiveness of these interventions.

200
Q

What is the primary purpose of an electrophysiology study (EPS)?

A. To evaluate heart valve function
B. To determine the source and treatment of dysrhythmias
C. To assess ejection fraction (EF)
D. To visualize coronary artery blockages

A

B. To determine the source and treatment of dysrhythmias

Rationale: EPS is primarily used to identify the source and determine the appropriate treatment for dysrhythmias, particularly in patients at risk for sudden cardiac death.

201
Q

How are the electrodes positioned during an electrophysiology study (EPS)?

A. By inserting them through a vein in the neck
B. By placing them externally on the chest
C. By inserting them into the heart chambers
D. By injecting them into the arteries

A

C. By inserting them into the heart chambers

Rationale: During EPS, electrodes are placed inside the heart chambers to record and manipulate the heart’s electrical activity, helping to assess the function of the SA node, AV node, and ventricular conduction.

202
Q

What is the primary risk for patients with symptomatic tachydysrhythmias?

A. Heart failure
B. Sudden cardiac death
C. Pulmonary embolism
D. Ischemic stroke

A

B. Sudden cardiac death

Rationale: Patients with symptomatic tachydysrhythmias are at high risk for sudden cardiac death, making accurate diagnosis and treatment critical.

203
Q

Nursing care after an electrophysiology study (EPS) is most similar to that of a patient after which procedure?

A. Coronary angiography
B. Electrocardiogram (ECG)
C. Chest x-ray
D. Cardiac catheterization

A

D. Cardiac catheterization

Rationale: Post-procedure nursing care after EPS is similar to that of a patient following a cardiac catheterization, with a focus on monitoring for bleeding and ensuring patient stability.

204
Q

What type of information does an electrophysiology study (EPS) provide about the heart?

A. Conduction patterns of the SA node, AV node, and ventricles
B. Coronary artery patency
C. Cardiac output and ejection fraction
D. Heart muscle oxygenation levels

A

A. Conduction patterns of the SA node, AV node, and ventricles

Rationale: EPS provides detailed information on the electrical conduction of the heart, including the SA node, AV node, and ventricles, to help assess and treat dysrhythmias.

205
Q

What is the primary purpose of cardiac catheterization with coronary angiography?

A. To measure ejection fraction and assess cardiac output
B. To determine the need for coronary artery bypass surgery
C. To assess electrical activity in the heart
D. To evaluate chest pain and assess heart function and major vessels

A

D. To evaluate chest pain and assess heart function and major vessels

Rationale: Cardiac catheterization with coronary angiography is used to evaluate chest pain, measure pressures within the heart, and assess the patency of the coronary arteries to identify any blockages or circulatory issues.

206
Q

What is a key nursing responsibility after a patient undergoes an electrophysiology study (EPS)?

A. Monitor for signs of coronary artery blockage
B. Encourage mobility to prevent blood clots
C. Administer antidysrhythmic medications as prescribed
D. Assess vital signs often and monitor ECG continuously

A

D. Assess vital signs often and monitor ECG continuously

Rationale: After an EPS, nurses need to monitor vital signs frequently and provide continuous ECG monitoring to ensure there are no complications, such as dysrhythmias or hemodynamic instability.

207
Q

What is the pre-procedure nursing care for a patient undergoing an electrophysiology study (EPS)?

A. Administer anticoagulants and keep the patient on bed rest

B. Encourage deep breathing exercises and administer IV antibiotics

C. Withhold antidysrhythmic drugs and keep the patient NPO for 6–8 hours

D. Check blood pressure and administer antihypertensive drugs

A

C. Withhold antidysrhythmic drugs and keep the patient NPO for 6–8 hours

Rationale: Antidysrhythmic drugs are often stopped several days before the EPS to ensure accurate test results. Patients should also be kept NPO for 6–8 hours before the procedure.

208
Q

What is the primary purpose of measuring Fractional Flow Reserve (FFR) during a cardiac catheterization?

A. To assess the heart’s electrical activity
B. To determine if angioplasty or stenting is needed for nonsignificant blockages
C. To evaluate the heart’s pumping efficiency
D. To measure oxygenation levels in coronary arteries

A

B. To determine if angioplasty or stenting is needed for nonsignificant blockages

Rationale: FFR helps assess whether angioplasty or stenting is needed for nonsignificant blockages in coronary arteries by measuring pressure and flow to determine the severity of the blockage.

209
Q

Which of the following is a nursing responsibility after a patient undergoes intravascular ultrasound (IVUS) during cardiac catheterization?

A. Monitor for signs of allergic reaction to the contrast media
B. Assess for fluid retention due to contrast dye
C. Encourage the patient to cough deeply
D. Inspect the insertion site for bleeding or swelling

A

D. Inspect the insertion site for bleeding or swelling

Rationale: After IVUS, which is done during cardiac catheterization, nurses should assess the insertion site for complications such as bleeding or swelling, just like other invasive catheterization procedures.

210
Q

What is an important nursing action before a peripheral arteriography procedure?

A. Administer IV sedatives to ensure relaxation during the procedure

B. Ensure the patient is NPO for at least 12 hours before the procedure

C. Assess for contrast media allergy and explain the sensation of the contrast injection

D. Instruct the patient to increase fluid intake immediately before the procedure

A

C. Assess for contrast media allergy and explain the sensation of the contrast injection

Rationale: Before peripheral arteriography, the nurse should assess for contrast media allergy and explain to the patient that they may experience a warm sensation or salty taste after the contrast is injected.

211
Q

What is the primary nursing action during peripheral arteriography?

A. Administer IV contrast media
B. Increase the patient’s IV fluid intake
C. Monitor the patient for allergic reactions to the contrast media
D. Ensure the patient remains NPO

A

C. Monitor the patient for allergic reactions to the contrast media

Rationale: During peripheral arteriography, the nurse should closely monitor the patient for potential allergic reactions to the contrast media. This is a key responsibility to ensure patient safety during the procedure.

212
Q

What is the most important nursing responsibility after peripheral arteriography?

A. Encourage the patient to walk to promote circulation
B. Inspect the insertion site for bleeding or swelling
C. Administer diuretics to prevent fluid overload
D. Give oxygen to enhance blood flow

A

B. Inspect the insertion site for bleeding or swelling

Rationale: After peripheral arteriography, the nurse must inspect the insertion site for complications such as bleeding or swelling and check the extremity for proper pulsation, warmth, and color.

213
Q

What is the expected sensation a patient might experience during contrast injection for a peripheral arteriography procedure?

A. A sudden sharp pain in the chest
B. A warm sensation and a salty taste
C. A feeling of dizziness and lightheadedness
D. Nausea and vomiting

A

B. A warm sensation and a salty taste

Rationale: Patients may experience a warm sensation and a salty taste immediately after the contrast injection during peripheral arteriography due to the contrast medium.

214
Q

What is the primary use of intravascular ultrasound (IVUS) during cardiac catheterization?

A. To evaluate heart rhythm
B. To assess the size, consistency, and location of plaque in the arteries
C. To measure oxygen levels in coronary arteries
D. To assess the electrical activity of the heart

A

B. To assess the size, consistency, and location of plaque in the arteries

Rationale: IVUS is used to assess the blood vessel patency, size, and consistency of plaque and evaluate the effectiveness of coronary artery treatments.

215
Q

Why are antidysrhythmic drugs withheld before an electrophysiology study (EPS)?

A. To ensure accurate electrical activity recording during the study
B. To increase the heart rate for the procedure
C. To decrease the risk of blood clots during the procedure
D. To prevent contrast-induced nephropathy

A

A. To ensure accurate electrical activity recording during the study

Rationale: Antidysrhythmic drugs are withheld before EPS to avoid interference with the heart’s electrical activity, ensuring accurate test results.

216
Q

Why might a pacemaker or implantable cardioverter defibrillator (ICD) be inserted during an electrophysiology study (EPS)?

A. To treat any acute myocardial infarction detected during the procedure
B. To correct electrical conduction issues or manage dysrhythmias
C. To perform angioplasty and treat blockages
D. To assist in coronary artery bypass surgery

A

B. To correct electrical conduction issues or manage dysrhythmias

Rationale: Pacemakers or ICDs may be inserted during EPS to correct electrical conduction problems or to manage dysrhythmias, especially if the procedure identifies a need for immediate intervention.

217
Q

Which of the following is a potential contraindication for peripheral arteriography?

A. Mild hypertension
B. Obesity
C. Non-urgent vascular issues
D. Severe renal impairment

A

D. Severe renal impairment

Rationale: Severe renal impairment is a potential contraindication due to the contrast media used in peripheral arteriography, which could further affect kidney function

218
Q

Which nursing responsibilities are priorities when caring for a patient returning from a cardiac catheterization? (select all that apply)

a. Monitoring vital signs and ECG

b. Checking the catheter insertion site and distal pulses

c. Helping the patient to ambulate to the bathroom to void

d. Telling the patient that he will be sleepy from the general anesthesia

e. Teaching the patient about the risks of the radioactive isotope injection

A

a. Monitoring vital signs and ECG
b. Checking the catheter insertion site and distal pulses

219
Q

A patient is being treated for heart failure. Which laboratory test result will the nurse review to determine the effects of the treatment?

a. Troponin

b. Homocysteine (Hcy)

c. Low-density lipoprotein (LDL)

d. B-type natriuretic peptide (BNP)

A

d. B-type natriuretic peptide (BNP)

Rationale: Levels of BNP are a marker for heart failure. The other laboratory results would assess for myocardial infarction (troponin) or the risk for coronary artery disease (Hcy and LDL).

220
Q

A patient is scheduled for a cardiac catheterization with coronary angiography. What information would the nurse provide before the procedure?

a. It will be important not to move at all during the procedure.

b. A flushed feeling is common when the contrast dye is injected.

c. Monitored anesthesia care will be provided during the procedure.

d. Arterial pressure monitoring will be needed for 24 hours after the test.

A

b. A flushed feeling is common when the contrast dye is injected.

Rationale: A sensation of warmth or flushing is common when the contrast material is injected, which can be anxiety producing unless it has been discussed with the patient. The patient may receive a sedative drug before the procedure but monitored anesthesia care is not used. Arterial pressure monitoring is not routinely used after the procedure to monitor blood pressure. The patient is not immobile during cardiac catheterization and may be asked to cough or take deep breaths.

221
Q

A patient will be evaluated for rhythm disturbances with a Holter monitor. Which instruction would the nurse provide?

a. Connect the recorder to a computer once daily.

b. Exercise more than usual while the monitor is in place.

c. Remove the electrodes when taking a shower or tub bath.

d. Keep a diary of daily activities while the monitor is worn.

A

d. Keep a diary of daily activities while the monitor is worn.

Rationale: The patient is taught to keep a diary describing daily activities while Holter monitoring is being accomplished to help correlate any rhythm disturbances with patient activities. Patients are taught that they should not take a shower or bath during Holter monitoring and that they should continue with their usual daily activities. The recorder stores the information about the patient‘s rhythm until the end of the testing, when it is removed and the data are analyzed.

222
Q

A patient who developed chest pain 4 hours ago may be having a myocardial infarction.

Which laboratory test result would be most helpful in indicating myocardial damage?

a. Troponins

b. Myoglobin

c. Homocysteine (Hcy)

d. Creatine kinase-MB (CK-MB)

A

a. Troponins

Rationale: Cardiac troponins start to elevate 4 to 6 hours after myocardial injury and are highly specific to myocardium. They are the preferred diagnostic marker for myocardial infarction. High-sensitivity troponin (hs-cTnT, hs-cTnI) assays provide even earlier detection of a heart event, within 1-3 hours. Myoglobin rises in response to myocardial injury within 30 to 60 minutes but is nonspecific and rapidly cleared from the body, limiting its use in the diagnosis of myocardial infarction. Creatine kinase (CK-MB) increases 4 to 6 hours after myocardial injury but is rarely used now for diagnosis of acute MI. Homocysteine (Hcy) is an amino acid that is made during protein catabolism. Elevated levels of Hcy are linked to a higher risk of CVD, peripheral vascular disease, and stroke.

223
Q

Which action will the nurse implement for a patient who arrives for a calcium-scoring CT scan?

a. Insert an IV catheter.

b. Instruct the patient to lie still.

c. Administer oral sedative medications.

d. Confirm that the patient has been fasting.

A

b. Instruct the patient to lie still.

Rationale: The patient should remain still during the scan. The procedure is rapid and involves little risk, so none of the other actions are necessary.

224
Q

Which information obtained by the nurse who is admitting the patient for magnetic resonance imaging (MRI) will be important to report to the health care provider before the MRI?

a. The patient has an allergy to shellfish.

b. The patient has a history of atherosclerosis.

c. The patient has a permanent cardiac pacemaker.

d. The patient took the prescribed heart medications today.

A

c. The patient has a permanent cardiac pacemaker.

Rationale: MRI is discouraged in those with older model pacemakers and ICDs because the magnets can change the function of the devices. However, when there is a strong clinical need and the benefits outweigh the risks, MRI can be done at centers experienced in this procedure. It will be important to determine the type of pacemaker. Many newer models of pacemakers and ICDs are approved for use with MRI.

225
Q

The nurse is monitoring a patient who is undergoing exercise (stress) testing on a treadmill.

Which assessment finding requires the most rapid action by the nurse?

a. Patient reports feeling tired

b. Sinus tachycardia at a rate of 110 beats/min

c. Inversion of T waves on the electrocardiogram

d. Blood pressure (BP) increase from 134/68 to 150/80 mm Hg

A

c. Inversion of T waves on the electrocardiogram

Rationale: ECG changes associated with coronary ischemia (such as T-wave inversions and ST segment depression) indicate that the myocardium is not getting adequate O 2 delivery and that the exercise test should be stopped immediately. Increases in BP and heart rate are normal responses to aerobic exercise. Feeling tired is also normal as the intensity of exercise increases during the stress testing.

226
Q

The nurse is admitting a patient for a cardiac catheterization and coronary angiogram. Which information is important for the nurse to communicate to the health care provider before the test?

a. The patient‘s pedal pulses are +1.

b. The patient is allergic to contrast dye.

c. The patient had a heart attack 1 year ago.

d. The patient has not eaten anything today.

A

b. The patient is allergic to contrast dye.

Rationale: Patients who have allergies to contrast dye will require treatment with medications, such as corticosteroids and antihistamines before the angiogram. The other information may be communicated to the health care provider but will not require a change in the usual pre-cardiac catheterization orders or medications.

227
Q

A transesophageal echocardiogram (TEE) is planned for a patient hospitalized with possible endocarditis. Which action included in the standard TEE orders will the nurse need to accomplish first?

a. Start an IV line.

b. Start O 2 per nasal cannula.

c. Place the patient on NPO status.

d. Give lorazepam (Ativan) 1 mg IV.

A

c. Place the patient on NPO status.

Rationale: The patient will need to be NPO for 6 hours preceding the TEE, so the nurse should place the patient on NPO status as soon as the order is received. The other actions also will need to be accomplished but not until just before or during the procedure.

228
Q

The nurse is reviewing the laboratory results for newly admitted patients on the cardiovascular unit. Which laboratory result is most important to communicate rapidly to the health care provider?

a. High troponin I level

b. Increased triglyceride level

c. Very low homocysteine level

d. Elevated C-reactive protein level

A

a. High troponin I level

Rationale: The elevation in troponin I indicates that the patient has had an acute myocardial infarction. Further assessment and interventions are indicated. The other laboratory results indicate increased risk for coronary artery disease but are not associated with acute cardiac problems that need immediate intervention.